RC Comp Exam

Pataasin ang iyong marka sa homework at exams ngayon gamit ang Quizwiz!

Compare and contrast a personality trait assessment and a personality disorder assessment. Provide an example of each. When is it appropriate to use a personality disorder assessment? What outcomes can you expect?

Personality Trait Assessment: help the counselor and client understand the client's attitudes, character traits, interpersonal needs, and/or think intrinsic motivation. ---Personality assessments have been used to identify psychopathology as well as strengths and weaknesses in occupational decisions. ---Ex of personality assessments: MMPI (Minnesota Multiphasic Personality Inventory) ---The most common assessment tool since counselor has the ability to directly assess a client's personality in Observation Personality Disorder Assessment: used when problematic, pervasive, and persistent personality traits are presented in their extremes. ---Ex of personality disorder assessment: Personality Assessment inventory (PAI) ------PAI is widely used for self-report, objective inventory used to assess psychopathology, some personality disorders, and interpersonal traits in adults 18 years and older. ------This assessment is helpful in providing DC information relevant to treatment planning

Informed consent and advocacy are two (2) aspects of the counseling practice that have widespread applicability in various counseling settings. Provide definitions of these terms and relate these practices to core ethical principles and relevant ethical standards. Finally, discuss ethical issues and considerations you would need to keep in mind to ensure that these practices are properly applied.

Advocacy: promoting the well-being of individuals, groups and the profession within systems and organizations. Advocacy seeks fair treatment and full physical and programmatic access for clients, and the removal of any barriers or obstacles that inhibit access, growth, and development (CRCC Code of Prof... pg. 37) -"Through advocacy, the counselor assists the individual to achieve her goals through participating in the individual's environments," (Strauser, 2014). - Career textbook -Advocacy counseling addresses external barriers that interfere with human development (Orientation to the Counseling Profession, pg ix) Informed Consent: a process of communication between the counselor and the clients that results in an authorization or decision by clients based upon an appreciation and understanding of the facts and implications of an action (CRCC's Code... p. 39) -Generally, informed consent occurs in the first session and as frequently as necessary thereafter to ensure that clients are aware of and understand their rights and responsibilities within the counseling relationship. Perhaps the most important issue covered in informed consent is confidentiality and its expectations. Advocacy + informed consent = rehab counselors obtain client consent prior to engaging in advocacy efforts on behalf of an identifiable client to improve the provision of services and to work towards removal of systematic barriers or obstacles that inhibit client access growth, and development. Rehab counselors are aware of and sensitive to the needs of IWD. Rehab counselors advocate at the individual, group, institutional, and societal levels to (1) promote opportunity and access; (2) improve the quality of life for IWDs; and (3) remove potential barriers to the provision of or access to services. Rehab counselors recognize that disability often occurs in tandem with other social justice issues (ex: poverty, homelessness, trauma). Ethical Concerns -Rehab counselors have an ethical obligation to actively participate in the process of promoting accessibility for IWD -Accessibility: a site, facility, work environment, service, or program that is easy to approach, enter, operate, participate in, and/or use safely and with dignity by an IWD -Counseling practice standard (C.2.a.) translates these obligations into action by directing counselors to "facilitate the provision of necessary accommodations, including physically and programmatically accessible facilities and services to IWD." -Barriers to access standard (C.2.b.) further required rehab counselors to "collaborate with clients and/or others to identify barriers based on the functional limitations of clients" and then communicate this information to the appropriate authorities "to facilitate removal of barriers to access."

Describe the licensure and credentialing bodies that govern the profession that you are about to enter. Carefully articulate a statement about your scope of practice and how to increase your professional expertise upon graduation.

Certification is through the CRCC: ^^^ in question 1 Scope of practice from internship/practicum; come from accredited programs. Increasing professional expertise can be done from attending workshops and classes that go towards CRC recertification, as well as frequently revisit the code of ethics.

Define "evidence-based practices." Make the case for rehabilitation counseling (RC students), mental health counseling (CMHC students), or school counseling (SC students) as having evidence-based interventions?

Evidence-based practices: the application of research-based treatments, into our daily care practices and clinical decision-making. Rehab counselors have a responsibility to the public to engage in practices that are based on accepted research methodologies and evidence-based practices, (Section D: Professional Responsibility, CRC Code of Ethics, p. 13). Prevents counselors from just doing whatever they want. Provides better protection for counselors in defending/documenting their clinical choices D.6 Scientific Bases for Interventions (CRC Code of Ethics, pgs 15-16): ---Acceptable techniques/procedures/modalities - RCs use techniques/procedures/modalities that are grounded in theory and/or have an empirical or scientific foundation. ---New or novel practices - when RCs use new or novel techniques/procedures/modalities, they explain any related potential risks, benefits, and ethical considerations. RCs work to minimize any potential risks or harm when using these techniques/procedures/modalities. ---Harmful practices - RCs do not use techniques/procedures/modalities when evidence suggests the likelihood of harm, even if such services are requested. ---Credible resources - RCs make reasonable efforts to ensure the resources used or accessed in RC are credible and valid (e.g., internet sites, mobile applications, books). Concerns about EBP: ---Lack of training ---Limited access to treatment manuals ---Inadequate research evaluation skills ---Limited professional supervision

Outline the stages of group counseling. For each stage discuss and provide an example of how relationships may manifest in group; your role as leader in each stage including task processing; and the overall goal of each stage of the group experience.

Forming: characterized by initial caution with any new experience. Members try to avoid being rejected by others, the leader, or even themselves. ---Goal: to build relationships within the group and clarify goals Relationship ex: members tend to be superficial and center convo around historical or future events that don't have direct impact on the group ------Manipulators, monopolizes, resisters, sarcastic, diagnostic analyzers, focus on others ---Task processing: 1. Dealing with apprehension 2. Reviewing member's goals and contracts 3. Clarifying group rules 4. Setting limits 5. Promoting positive interchange ---Leader's role: 1. Joining - intros, ice breakers 2. Linking - connecting members 3. Drawing out - inviting silent members 4. Cutting off - stop rambling or doorknob statements 5. Clarifying - restating purpose 6. Assessing - afterwards Storming: a time of conflict and anxiety in a group when it moves from primary tension (awkwardness about being in a strange situation) to secondary tension (intragroup conflict). It's a period when group members and leaders struggle with issues related to structure, direction, control, catharsis, and interpersonal relationships. ---Goal: overcome obstacles, accept individual differences, and work through conflicting ideas on group tasks/goals ---Relationship ex: group members are initially more anxious in their interactions with each other. Some avoid talking, others will be open and assertive. The concern for power is relevant and the attitude group members feel about who is in power is relevant. Members' attitude about trusting the group and its leader are an issue ------Tension, intragroup flexing, threshold for frustration and sensitivity, elements of power and control, resistance to change ---Task Processing: Appears to regress during this stage because there is a great deal of attention on personal matters and not the objectives of the group. ---Leader's role: ---------Process observer - a 3rd party professional who observes the group and gives it feedback on its interpersonal and interactive process. Helps members work through their feelings. ----------Leveling - process in which group members are encouraged to interact freely and evenly with each other. In leveling, group members who are under-participatory are drawn out and those who are excessively active are helped to modify their behavior. -----------Feedback - one individual giving another their perception of a behavior, sharing relevant information with other people, such as how they appear to others, so they can make decisions about whether they would like to change. -----------Motivational interviewing - a client-centered, directive method for resolving ambivalence that impedes change. Techniques include open-ended questions, intentional reflections, recognizing readiness, and guiding individuals in setting personal goals. Norming: process in which members form an identity as a group and a sense of "we-ness" prevails; there is enthusiasm, cooperation, and cohesiveness currently. In many ways, the norming stage parallels the forming stage regarding its emphasis on positive feelings. Norming, like storming, lasts for only a few sessions; it sets the pattern for the next stage: performing (i.e., working). ---Goal: shift energy to group's goals and show an increase in productivity ---Relationship ex: members have more positive attitude toward others and the experience itself. A sense of belongingness; willing to give of themselves and committed to taking needed actions. Peer interactions are manifested through ---Identification - "normal" development process in which individuals see themselves as being similar to one another ---Here and now experience - dealing with immediate feelings and interactions (existential variables) ---Hope - cognitive (belief that what's desire is possible, and things will turn out for the best) and emotional level (the feeling that what you wish for will occur) ---Cooperation - working together for a common purpose or good ---Collaboration - ^~ members who think they can work in a harmonious, cooperative manner are likely to share facts and feelings about themselves and other matters with the group ---Cohesion - sense of "groupness" or "we-ness" ---Task processing: ----------Establish norms (prescriptive (dos) and proscriptive (don'ts)) or rules and standard from which to operate the group. Members learn to regulate, evaluate, and coordinate their actions. ------------Commitment to the group as a while and its rules as well as to individual goals ---Leader's role: -----------Supporting (validation, encouragement) -----------Empathizing (understanding another's subjective perception) -----------Facilitating (guidance, clear communication) ----------Modeling (self-disclosure, norms) Performing: most unified and productive stage that focuses on the achievement of individual and group goals and the movement of the group itself as a system. ---Goal: work on and achieving goals ---Relationship ex: most members appear to show genuine concern for each other on a deep, personal level. Interpersonal bonding (cohesiveness) usually increases. Emotional closeness likely to grow. Self-disclosure is multidimensional (involves listening and receiving feedback as well as speaking). Group members become increasingly aware of individual participation, there is more sharing and risk taking. ---Task processing: productivity is emphasized by focusing on improving themselves or achieving specific individual or group goals. --------Rounds - equal sharing time for members --------Role-playing - given a chance to assume role that differs widely from their present behavior. Tool for bringing patterns of behavior and their consequences into focus. Need trust and caring to do this. --------Homework - working outside of the group itself --------Incorporation - personal awareness and appreciation of what the group has accomplished on both and individual and a collective level. ---Leader's role: ----------Modeling Exercises - less direct showing & more experiential integration --------------------To increase comfort level, provide leader with useful info, generate discussion and focus, shift focus, deepen focus, provide opportunity for experiential learning, and provide fun and relaxation ----------Group observing group - group breaks into two smaller groups and each observes (as outsiders) the other function for a set amount of time; aka fishbowl observation ----------Brainstorming - initial generation of ideas in a nonjudgmental manner ----------Nominal group technique (NGT) - 6-step process involving generation of a number of ideas/solutions connected with a problem statement. Doesn't require the open exposure of members as mush as brainstorming and ends with a vote, discussion, and re-vote on priorities for resolving a situation. (45-90 minutes long with people of diverse settings). ----------Synthetics - integration of diverse individuals into a problem-stating, problem-solving group ----------Written projections - imagine seeing self or group successful in future and share what the experience was like ----------Group processing - such as note taking to prevent memory decay of the group experiences ----------Teach skills Closing: members coming to know themselves on a deeper level; transition with summarization and reassurance. ---Goal: reassuring transition ---Relationship ex: feelings of empathy, compassion, care; members relate to each other on a deep, emotional level; feelings of warmth and sorrow ---Task processing: ----------Reflection ----------Processing memories ----------Evaluation ----------Acknowledge of feelings ----------Reassurance - "what now?" ---Leader's role: ----------Boundaries (time limit established at the beginning of group) ----------Capping skills --------------------Review and summarization --------------------Assessing growth (reinforce progress) --------------------Finishing business - resolution --------------------Implementing cognitive decision making (real world application) --------------------Feedback --------------------Goodbyes --------------------Planning for the future ------------------------------Provide resources ----------Focus on needs of group to terminate --------------------Reflection ------------------------------What was most helpful? Learned? What will be taken with them from group?

Joe is 21 years old and has recently come to you for career counseling as he feels that he is "lost" career-wise. He would like to go back to school and get a degree "in something." Describe a career counseling strategy and specific assessments that you will implement with Bob. What outcomes can he expect?

I would use motivational interviewing and the stages of change model to find out more. The four steps of the MI process are engage, focus, evoke, and plan. Compassion and empathy are foundational in MI. First, I would engage Joe using open-ended questions, empathic reflections, autonomy-supportive affirmations, and rolling with resistance. This is to build rapport, gain a deeper sense of understanding, and learn more about the JC' background. Get an understanding of the "lost feeling", interests, strengths, limitations, etc. Second, Focus on understanding what Joe is saying and how he feels. Empathize with his perspective. Focus by reflecting, summarizing, and developing discrepancies. To be an effective guide, you need to know where you're going. Focus on what he has done in the past and ideas/goals of the future. The next step is to respectfully evoke your client's thoughts, feelings, motivation, and concerns. That is, get the discussion going and directed at underlying concerns that might increase or decrease motivation in certain job areas. Take a career interest inventory survey to gain more info on areas that Joe might succeed. Talk through them and listen to clients' reactions. The last step is to talk about a plan. Might be another interest inventory, job shadows, informational interviews, etc. In some cases, the plan may just be to talk about it again later if the person isn't ready. The stages of change model is as follows: Precontemplation (Not yet acknowledging that there is a problem behavior that needs to be changed) Contemplation (Acknowledging that there is a problem but not yet ready or sure of wanting to make a change) Preparation/Determination (Getting ready to change) Action/Willpower (Changing behavior) Maintenance (Maintaining the behavior change). Joe is in the contemplation stage, as he is aware that change needs to occur, but has not committed to the process. He is gathering information to learn more about what he has to do to not feel "lost" at work. In this stage, people are on a teeter-totter, weighing the pros and cons of quitting or modifying their behavior. Although they think about the negative aspects of their bad habit and the positives associated with giving it up (or reducing), they may doubt that the long-term benefits associated with quitting will outweigh the short-term costs. If Joe decides to continue to work with me and take active steps towards bettering his career, then I would expect success. However, Joe has to want to make a change and commit to it for success to come.

You are about to discuss the results of a career interest inventory with your client (or student). What would your client (or student) need to know about the limitations of career-related tests before an interpretive discussion begins?

Limitations: --Client's reflection on results can provide more answers than the test itself --Life experiences influence testing --Test conditions affect results (ex: home vs. in busy waiting room) --Multipotentiality - like everything, good at everything --Expectancy effects - expect a certain result, will answer to get that result --Test bias (ex: women score higher on social) --Cultural considerations --Assessing values vs. skills vs. interests --------Test is just one piece of evidence. Combine with other pieces of evidence for better/more accurate results Explain beforehand: --Even good assessments make mistakes --What the test intends to measure --Reflection can provide better clarity than results sometimes --Cultural considerations

Professional disclosure statements are recommended to assist counselors in complying with the ethical obligation of providing informed consent to clients. First, define both of these terms and describe their relationship to one another. Next, discuss the necessary legal elements for informed consent. Finally, provide a rationale for using a professional disclosure statement, describe how it should be used, and the components that should be included in such a statement.

Professional disclosure statements: the rights and responsibilities of both the counselor and the client that is to be written and also reviewed orally. These are presented in a manner best suited to the needs of the client. Disclosure at the outset of the professional relationship minimally includes: (1) qualifications, credentials, and relevant experience of the counselor; (2) purpose, goals, techniques, limitations, and the nature of potential risks and benefits of services; (3) frequency and length of services; (4) confidentiality and limitations regarding confidentiality (including how a supervisor and/or treatment team professional is involved); (5) contingencies for continuation of services upon the extended absence, incapacitation, or death of the counselor; (6) fees and/or payment arrangements; (7) record preservation and release policies; (8) risks associated with electronic communication; and (9) legal issues affecting services. When necessary, counselors disclose other information consistent with the organization and/or employer policies or legal requirements. Counselors recognize that disclosure of these issues may need to be reiterated or expanded upon through the professional relationship. (CRCC's Code of Professional Ethics for Rehabilitation Counselors, p. 8) Informed consent: "Section A.2.b specifies that informed consent includes, but is not limited to, "purpose, goals, techniques, procedures, limitations, potential risks, and benefits of services; the counselor's qualifications, credentials, and relevant experience, and the use of technology; . . . the intended use of tests and reports, fees, and billing arrangements"; the right to confidentiality and limitations; the continuation of services should the counselor become incapacitated; "obtaining clear information about their records"; participating in ongoing treatment planning; and the right to refuse treatment at any time along with the potential consequences for doing so. In addition, the code addresses the need to balance assent form minors and others incapable of giving consent without the assent of parents and family members who hold the legal rights of consent, protection, and decision making on their behalf," Erford 2018, pg. 74). -"... many state counseling boards now require written consent so clients can choose a counseling relationship with a qualified provider from an informed consumer position," (Erford, 2018, p. 74) -Another definition: a document a client signs acknowledging that they are aware of the session activity in which they are about to participate and is doing so voluntarily (Group Textbook, glossary) ; document informing clients asap in the therapeutic relationship about the nature and anticipated course of therapy, fees, involvement of third parties, and limits of confidentiality (APA, 2002, p. 1072 from Counseling Psychology 3rd edition, pg.57) -Statement of purpose, leader/counselor's policies, ground rules, and full discussion of the limits of confidentiality (Counseling Psychology, 465). Relationship -Informed consent is addressed within PDS -Is an on-going process (reminders and readdressing when appropriate or situation relates to it) Allie's email screenshots: -Informed consent -Verbal -For it to be informed consent, client must first have a clear understanding of the relevant facts, risks, and benefits and available alternatives provided. -Requires legal documentation in all 50 states -Originated from recognition of individuals having rights to freedom, autonomy, and human dignity - I think this is about just research consent— Voluntary consent rules: could only be requested for experimentation for the benefit of society, potential acquisition of understanding of the pathology of disease, and for studies performed that avoided physical and mental suffering. -Most relevant in psychotherapy, counseling, assessment, and research settings Disclosure statements Written

Outline a theory of group counseling; including the premise of the theory, the role of the leader, desired outcomes as well as strengths and limitations. Describe three (3) techniques associated with a particular group counseling theory. How might these techniques depend on the specific stage of the group process and the developmental level of the participants?

Solution-Focused (individual therapeutic work in group setting) Premise: focus on strengths, skills, and resources. Identify problems and focus on solutions (defined prob should be achievable). Present-focused. Any amount of change - progress. Leader's Role: optimism (instill hope), encourage change (small & measurable), focus on possibilities, emphasize internal resources, reframe Desired outcomes: change way of thinking about their lives. Instead of seeing life problematically, group members may recognize that although difficulties occur, there are exceptions. Strengths: --They concentrate on & are directed by group members' situations --Group leaders assist members in defining their situations with clarity, precision, and possibilities --Any change is reinforced --Doesn't focus on clinical understandings of members' situation --Empowers group members to assess and utilize their resources --Flexibility in working with a variety of situation Limitations: --Little attention paid to members' history, and sometimes people need to resolve issues from their past --It ends when a behavioral goal is reached --Unless there's careful co-creation of problem definitions, it can cause leaders and members to focus on the presenting problem and miss issues of greater importance. Techniques: --Miracle Question: "If a miracle happened to where you woke up tomorrow and the problem was solved, what differences would you notice (in self and circumstances)?" -----------This is useful in the forming stage by helping to identify goals/desired outcomes. --Scaling questions: questions asked on a 1 to 10 scale to help move members toward their goals by asking them what would move their rating to the next number (ex. "what would bring you from a 6 to a 7?) -----------This is useful in the performing stage as progress toward goal achievement is being made. --Reframing: changing a perception by explaining a situation from a different context (ex. Rob might think Shani doesn't like him because she never says hi or make eye contact with him. Reframe: Shani is shy and doesn't initiate contact with anyone.) -----------This is useful in the forming, storming, and norming stages as group members change their perceptions from problem-focused to solution-focused and while members are trying to find their place/where they fit into the group.

Compare and contrast a developmental model of career development with a trait model of career development. Describe how each might apply to a person with a congenital physical disability that limits mobility.

Super's model of career development is a developmental model while Holland's theory is a trait model. Super focuses on developing a self-concept over time to become more realistic and stable, which then will allows a career choice to become more realistic and stable. This model uses work satisfaction to determine how effective it is for an individual to implement a self-concept into their work. Holland uses personality traits to determine best suitedness for work. For Holland, job satisfaction is determined based on how well one's personality fits with their job environment. If congruence is low, then their personality does not fit that job. Super believes that career congruence develops and improves over time, while Holland's theory states that congruence is dependent on personality and will not improve over time if incongruence exists between type and career environment. Both models take work satisfaction into account. Holland's Theory does possess aspects of a developmental model, but it more recognized as a trait theory. Under Super's model, an individual with a congenital physical disability that limits mobility would be able to grow over time in their career just as they are growing personally with their disability. Starting with the growth stage, the individual would develop their own self-concept. Once a self-concept has been developed, it will become more realistic as vocational preferences and considered in the exploration stage. In the establishment stage, tasks are secured that the individual will consider to be their own personal niche and they continue to grow on that. The maintenance stage preserves the work they have made up until that point as they move into the decline stage and lean toward retiring their major tasks. Disability is welcomed in this model and incorporated in their self-concept. Holland's theory might conclude that this individual would not have a realistic personality as mobility is limited and they may struggle working with their hands or in other areas that require movement. Because Holland's model can be affected by social aspects and environment, the way this individual grew up may have altered their behaviors. Growing up with a physical disability, their parents may have only allowed certain choices regarding preferences in activities, sports, interests, etc. This impacts their personality and what career choice they may be most compatible with.

Describe a current "best practice" supported by research in rehabilitation counseling. Describe the research that supports the practice as effective. How does the practice impact the outcome of counseling for consumers?

Trauma informed care exposure to abuse, neglect, discrimination, violence, and other adverse experiences increase a person's lifelong potential for serious health problems and engaging in health-risk behaviors, as documented by the landmark Adverse Childhood Experiences (ACE) study.1,2,3 Because of the ACE study, and other subsequent research, health care policymakers and providers increasingly recognize that exposure to traumatic events, especially as children, heighten patients' health risks long afterward. SAMHSA has incredible resources, research, and peer-reviewed articles supporting and educating a wide variety of professionals in trauma-informed care. TIC's values align closely with rehabilitation counseling, as both emphasize client involvement/client-directed services, empowerment/self-advocacy, and a strength's-based orientation. TIC is not a singular intervention, but a process; providing TIC doesn't require the counselor to be aware that the client has a trauma history. The fundamental principles to guide the process of change when utilizing TIC are safety, trustworthiness, choice, collaboration, and empowerment. In 2017 a sixth principle of cultural, historical, and gender issues was added, which acknowledges that culture, history, and politics affect every aspect of a trauma-informed approach. Utilizing TIC requires the counselor to be aware of the prevalence of trauma, understand how trauma impacts an individual's functional capacities, and have the ability to recognize other beneficial resources (mental health, etc.) By employing specific tenants of trauma-informed care, infused with traditional rehabilitation practice, rehabilitation counselors are in a unique position to comprehensively address consumers' needs and ultimately foster more constructive outcomes.

Joan was injured at work due to an unforeseeable accident and completed the paperwork for Workers' Compensation (WC) 10 days ago. She is receiving treatment and is expected to go back to work in 10 days to her current job, at same pay, with no permanent injury and no loss of body parts and functions. She saw an advertisement on late night TV suggesting that she could hire an aggressive law firm and obtain a large settlement. Joan has come to you for professional advice. In detail, explain: 1) what WC is; 2) who is eligible for WC; 3) what benefits go with WC and how they are determined; 4) who provides medical care under WC; and 5) how long benefits would continue under WC. Finally, provide Joan with your best consultation as to her choices at this time.

WC provides certain benefits to employees who receive injuries (85), occupational disease (85A) or occupational hearing loss (85B) arising out of and in the course of their employment. Benefits are payable regardless of fault and are the exclusive remedy of the employee against the employer. Nearly all employees who have work-related injuries in Iowa are eligible for Iowa workers' compensation benefits. While there are 10 very specific exemptions to those who can get WC, 7 of them can be covered if the employer has a workers comp insurance policy. --Medical Benefits: Your employer must pay for all reasonable and necessary medical care required to treat your injury. This includes reasonable and necessary travel expenses for treatment. --Disability benefits: TTD, HP, PPD or PTD benefits are paid at a weekly workers' compensation rate considering your marital status and number of exemption -----Temporary Total Disability(TTD): When you are off work more than three calendar days on account of injury, you may be entitled toTTD benefits beginning on the fourth day and continuing until you return to work or are medically recovered enough to return to similar work, whichever happens first. -----Temporary partial disability (TPD): If you return to work at a lesser paying job because of the injury, you may be entitled to benefits. -----Permanent partial disability (PPD): When your work injury results in a permanent impairment to your body, a permanent restriction, or an inability to earn wages similar to those earned before your injury, you may be entitled to PPD benefits. PPD benefits are in addition to healing period benefits -----Permanent total disability (PTD): If your work related injury leaves you incapable of returning to any type of wage earning employment, you may be entitled to permanent total disability benefits during that time when you cannot return to any gainful work. -----Healing period (HP): You may be entitled to HP benefits while recovering from an injury which produces a permanent impairment. No waiting period applies to HP benefits. These benefits begin on the first calendar day after the date of injury and continue until the first of the following occurs: • You return to work • You have recovered as much as anticipated from the injury • You are medically capable of returning to the same kind of work you did when injured Second injury fund benefits: If you have had a permanent disability to a hand, arm, foot, leg or eye and then have a job related injury that results in permanent partial disability to another hand, arm, foot, leg or eye, you may be entitled to "Second Injury Fund" benefits. Vocational rehabilitation benefits: You may be entitled to payment of $100.00 per week for up to 13 weeks if you are actively participating in a vocational rehabilitation program in order to make it possible for you to return to gainful employment after your injury. Death benefits: If you were dependent on someone who died as a result of an on the job injury, you may be eligible to receive death benefits. A surviving spouse may receive death benefits for life or until remarriage. Dependent children are entitled to death benefits until age 18 or, if actually dependent, age 25. The employer has the right to choose the medical care and must provide medical care reasonably suited to treat your injury. If you are dissatisfied with that care, you should discuss the problem with your employer Benefits continue until you can gainfully return to work or you are medically capable of completing the job you did when you got hurt. I cannot provide the client with legal advice, as it is out of my scope of practice. What I can do is inform her on how workers comp works and to strongly encourage her to consider if all the fees required to take on this task would be worth the effort.

Describe those features of the practice of rehabilitation counseling that make it a profession. Discuss at least two (2) professional groups that represent the interests of counselors and discuss at least one (1) code of professional standards that governs the practice of counseling.

A. Why are we different than anybody else? Specialists in our field: employment is everywhere, what does that employment look like, staying in our scope of practice for referral, contact agencies, advocacy; thinking about things applicable in our field; accredited. "Our scope of practice, accreditation standards, and certifications requirements support our identity as counselors. With our counseling skills as well as disability and rehab expertise, we also perform functions such as case management, advocacy, and consultation when working with out clients in achieving their individualized goals..." p. 20 "In considering rehab counselors within the broader context of counseling, rehab counselors could be considered counseling specialists who provide services to persons with disabilities (specialists because of the population they serve) and counseling specialists who work in rehab agencies or settings (specialists because of the setting in which they work). B. Why we are specialists in the counseling profession. P. 21 "The Council on Rehab Education (CORE) offers the following statement regarding rehab counselor, indicating rehab counselors are specialists within the field of counseling, "working directly with a disability or their advocates (ex: IVRS, SSA, SSDI, therapist), a rehab counselor is a special type of professional counselor who helps evaluate and coordinate needed services to assist people with disabilities...." (Council on Rehab Education, 2010). P. 21 "The Commission on Rehab Counselor Certification (CRCC, 2010) has identified rehabilitation counselors as counselors who possess specialized knowledge, skills, and attitudes—words that appear to acknowledge the specialty nature of rehab counseling within the counseling profession." C> Professional groups (2): ARCA: American Rehab Counseling Association From website: ARCA is an organization of rehab counseling practitioners, educators, and students concerned with improving the lives of people with disabilities. Rehab counselors are counselors with specialized training and expertise in providing counseling and other services to IWD. Its mission is to enhance the development of IWD throughout their lifespan and to promote excellence in the rehab counseling profession. ARCA's goal is to provide the type of leadership that encourages excellence in the areas of rehab counseling practice, research, consultation, and professional development. ARCA is equally interested in eliminating environmental and attitudinal barriers so that more opportunities are available with regard to education, employment, and community activities to IWD. These goals are addressed by ARCA through public education and legislative activities. One of ARCA's primary goals is to increase public awareness of rehab counseling and to extend its influence by encouraging members to become involved in the association's outreach and educational efforts. Other goals are to help members develop their leadership skills through participation in ARCA's organizational activities and to work with state officials to develop appropriate licensure requirements. Why you're in school / why some classes are accredited, and some aren't. CACREP: (new, improved edition of CORE) from CACREP.org: The vision of CACREP is to provide leadership and to promote excellence in professional preparation (ex. Schooling) through the accreditation (our program is accredited) of counseling and related educational programs. As an accrediting body, CACREP is committed to the development of standards and procedures that reflect the needs of a dynamic, diverse, and complex society. (Accreditation instils consistency throughout incoming professionals) CACREP is dedicated to: Encouraging and promoting the continuing development and improvement of preparation programs (preparation programs = ex. UI rehab couns program, improved development of program curriculum. Remember: tho there's a sense of consistency, there's always a need for improvement. CACREP recognizes the importance to study incoming professional's success or lack thereof and ways to implement change prior to beginning professional careers. THINK - this is why some students in today's age probably have to take different classes than those from say 5-10 years ago) and Preparing counseling and related professionals to provide services consistent with the ideal of optimal human development. (As an ex. Multiculturalism factors, cultural humility, new age identity groups such as LGBTQ+; newly identified/understood disabilities). CACREP maintains collaborative relationships with other groups that focus on accreditation, licensing, certification, and the professional development of counselors and related practitioners. The mission of CACREP is to promote the professional competence of counseling and related practitioners through: the development of preparation standards the encouragement of excellence in program development the accreditation of professional preparation programs D. A code of professional standards: CRCC (The Commission on Rehab Counseling Certification) The CRCC is the world's largest rehab counseling organization dedicated to improving the lives of individuals with disabilities. CRCC sets the standard for competent delivery of quality rehab counseling services through its nationally accredited and internationally recognized Certified Rehab Counselor (CRC) certification program and serves all rehab counseling professionals through the CRCC COMMUNITY. (This code/standard is stating that rehab counselors must be both accredited (UI Program) and certified (CRC exam) in order to abide by the code of professional standards) CRCC is the world's largest rehab counseling organization with nearly 15,000 current CRCs. (The majority of this profession has agreed to this standard given the number of participants that have agreed to this - further proving why it is an accredited certification and most agreed upon/accepted standard to follow (it's accredited through the NCCA) this is why the number 15,000 is important to remember) The CRC certification program is accredited by the National Commission for Certifying Agencies (NCCA).

Describe one (1) model of racial identity development including theorist, key constructs, and applicability in counseling clients from diverse backgrounds.

Critical Race Theory - examine society and culture as they relate to categorizations of race, law, and power Theorists: Derrick Bell, Patricia Williams, Richard Delgado, Kimberle Crenshaw, Camara Jones, Mari Matsuda Constructs: --Racism is engrained into American society --Institutional racism, not just indiv, are pervasive and dominant --Power structures are based on white privilege and white supremacy, perpetuating people of color --White supremacy and racial power are maintained over time --The law plays a role in keeping whites in power --Possible to transform the relationship between racial power and the law --Racial emancipation and anti-subordination can be achieved --Formulates why POC are more likely than whites to see American society as racist and be pessimistic about eliminating racism (power and privilege) --Articulates a more expansive theory of race and racism apart from intentional prejudice (e.g., microaggressions) --Moves beyond traditional Black/White paradigms in study of race relations (multiple races included) --Understands how racism intersects with other forms of oppression (gender, class, sexuality) Premises: --Systemic racism is pervasive --Racism is permanent --Racism must be challenged --Minority groups are subordinate to majority groups in power and privilege --Whiteness = normative in the US (interest convergence) --------White will support social justice to the point that it will benefit them --US law defines and affirms whiteness identity, privilege, and property --Identity and perceptions of identity can grant or deny resources Applicability to counseling diverse clients: --Use concepts from CRT to address racial disparities when working with POC --Use with cross-cultural counseling --Acknowledge that me as a light-skinned counselor can be perceived as the oppressor ------Consider worldview/culture when attempting to build rapport ------Broaching - opening our differences up to discussion ------Working alliance and advocacy ------Explore client self-identification ------Reflect on personal biases ------Use narrative storytelling for describing experiences with racism --Use CRT to acknowledge own privilege when working with POC culturally appropriate treatment approaches: Trauma informed care. CRT focuses on addressing racism, white supremacy, and white hegemony through social, structural, and legal systems. The bills and bans of CRT are political acts of aggression intend to silence, erase, and suppress BIPOC and LGBTGEQ+ history and experiences within classrooms and, ultimately, the United States. Trauma intersects in many different ways with culture, history, race, gender, location, and language. Trauma-informed systems acknowledge the compounding impact of structural inequity and are responsive to the unique needs of diverse communities, while also encouraging cultural humility from the professionals who utilize it.

Networking is essential for building a community that welcomes applicants with different abilities. Describe effective methods of marketing rehabilitation services and building employer relationships that lead to success in developing positive employment outcomes for persons with disabilities.

Employer Relations: -Effective business engagement: Approaching employers as high-impact strategic partners means looking beyond the immediate needs of a program or college and seeking ways to make local employers or industries competitive. It requires building ongoing opportunities for problem-solving and program development. Doing so requires approaching employers in a listening rather than an "asking" mode: less "What can you do for us?" and more "Where is your pain? How can we help in addressing your challenges?" effective business engagement strategies are mutually beneficial for both the supply and demand sides. The dual customer approach: In recent years, some states have recognized the value of seeing the employer as a customer, as much as the person with a disability. This is called the "dual-customer" approach. Really, this is about trying to identify all the needs that employers have, and being able to set up systems and services that respond to them. They view the employer as someone who needs to learn about individuals with disabilities, their accommodation requirements, and how to provide those accommodations. With this in mind, the employment professional can meet all those needs, thus helping both the job seeker and the business where they eventually work. Info on what RC is: -Cost and benefit of hiring IWDs (lect 6, slide 2) --Cost of accommodation -----About three-quarters (74.1%) reported a First Calendar Year Cost of $500 or less; more than three-quarters reported there were no indirect costs associated with the accommodation (ex: losing time because of training, supervisor's time, loss of production) --Benefits ----87.1% retain, 16.7% hire, or 11.5% promote a qualified or valued employee ----Almost three-quarters (73.8%) reported that the accommodation increased the affected employee's productivity ----Over half (50.5%) reported it increased the accommodated employee's attendance ----Other common direct benefits reported include 41.8% saving on worker's comp and other insurance, and 43.8% increased the diversity of the company Indirect Benefits -Increased overall company morale (60.7%) -Increased overall company productivity (57%) -Improved interactions with customers (42%) -Increased workplace safety (42.3%) -Increased overall company attendance (36%) Employer's Perception Ability to retain quality employees Increase company profitability An avoidance of costs associated with hiring and training a new employee Improve organizational culture and climate Consumer's Attitudes Toward Companies -Foster a sense among all employees that employers recognize both the value of the individual worker as a human being, and the inherent social benefits of creating and sustaining an inclusive workplace Legal benefit Legal tax breaks

Define forensic rehabilitation. Describe venues a rehabilitation expert may be called to testify and roles that a forensic rehabilitation counselor may fill.

Forensic rehabilitation is the evaluation of vocational and rehabilitation needs of individuals with disabilities in different legal settings. Expertise in the following fields are needed for forensic rehabilitation to be effective: worker's compensation, SSDI, VA, civil litigation, and others like jury selection, contested separation, and wrongful termination. A forensic rehabilitation counselor may serve as an expert witness to determine vocational damages like loss of capacity to earn, loss of opportunity to be employed, and future medical care.

What does the term scope of practice mean and what are the core aspects of scope of practice for rehabilitation counseling? How does your individual scope of practice relate to that of the profession and what ethical considerations must be taken into account to define the individual scope of practice? Finally, what do the ethical standards of the American Counseling Association (ACA)'s, American Mental Health Counseling Association (AMHCA)'s and Commission on Rehabilitation Counseling Certification (CRCC)'s Codes of Ethics require if a counselor wishes to add new areas of practice to his or her scope of practice?

Scope of practice is referred to as the extent and limits of activities considered "acceptable" by the individuals licensed or certified in a profession or specialty. The core aspects of rehabilitation counseling's scope of practice is to assist individuals with physical, mental, cognitive, developmental, and emotional disabilities to achieve their career, personal, and independent living goals in the most integrated setting possible though the application of the counseling process. This counseling process involves communication, goal setting, and beneficial change or growth in the client through self-advocacy, vocational, psychological, social, and behavioral interventions. My individual scope of practice is based on my own knowledge (such as having certification in Resiliency and trauma informed perspectives, supported employment, trained to use diagnostic instruments, using person-centered, motivational interviewing, and cognitive behavioral theoretical approaches). My scope of practice is narrower than that of the profession's as I am quite inexperienced and have work in a limited number of roles that are available to rehabilitation professionals. Ethically, rehabilitation counselors are bound to their individual scope of practice as to not overreach and be in ethical violation of using unfamiliar instruments/practices or advertising services that one has not been trained to provide. When a counselor wishes to add new areas to their scope of practice, they must increase their competence in order to both understand the requirements of a new discipline/practice and perform those tasks and functions. However, without the proper training, supervision, and/or professional license, you may find yourself navigating murky ethical waters. (https://crccertification.com/crcc-euniversity-1/ethics-courses/)When thinking about the differentiation between the rehabilitation counseling scope of practice and my individual scope of practice, it is of extreme importance to be aware of my own knowledge, skills, and abilities that I have gained so far through experience and education. It is my ethical responsibility to demonstrate awareness of this and only practice within the areas I have been trained and I am knowledgeable, ensuring that nonmaleficence is upheld. For example, it would be unethical for me to provide substance use treatment services to individuals experiencing substance use disorders without proper supervision, as this is a population in which I have little professional experience with. In addition, it would also be unethical to try and practice DBT without required training. Understanding and establishing boundaries of my individual scope of practice by being aware of the extent of my knowledge, experience, abilities, and skills ensures that services remain within the scope of practice -never extending beyond into areas in which I have not been trained. If counselors wish to add to his or her scope of practice they must increase their competence to understand the requirements of the new area and perform those tasks and functions. Proper training, supervision, and licensing (if applicable) needed might involve continued education courses and training.

What are the benefits of work beyond income? Are there consumers for whom work activity would not be a desirable rehabilitation goal? What is the counselor's ethical responsibility to consumers who are seeking employment?

1. Socialization 2. Community engagement 3. Consistency 4. Intellectual challenge 5. Self worth/identity 6. Passion / to make a difference 7. To gain perspective If the client is coming to vocational rehabilitation, some form of work activity is a necessary goal. If someone has an active substance abuse issue, work activity would not be an immediate goal. There would be other interventions and steps necessary before being able to place them in a job. If a student was transitioning out of high school with a desire for post-secondary education, work activity would not be a desirable goal either. Typically, rehabilitation likes to focus on one step at a time in order to set clients up for success. The only exception to this is the independent living program. The target population are individuals with significant disabilities who experience barriers to independent living in their home and communities and who also do not desire employment as a goal. A counselor's ethical responsibility to consumers seeking employment are: • respecting human rights and dignity; • ensuring the integrity of all professional relationships; • acting to alleviate personal distress and suffering; • enhancing the quality of professional knowledge and its application to increase professional and personal effectiveness; • promoting empowerment through self-advocacy and self-determination; • appreciating the diversity of human experience and appreciating culture; • emphasizing client strengths versus deficits; • serving individuals holistically; and • advocating for the fair and adequate provision of services. These values inform principles. They represent one important way of expressing a general ethical commitment that becomes more precisely defined and action-oriented when expressed as a principle. The fundamental spirit of caring and respect with which the Code is written is based upon six principles of ethical behavior: -Autonomy: To respect the rights of clients to be self-governing within their social and cultural framework. -Beneficence: To do good to others; to promote the well-being of clients. -Fidelity: To be faithful; to keep promises and honor the trust placed in rehabilitation counselors. -Justice: To be fair in the treatment of all clients; to provide appropriate services to all. -Nonmaleficence: To do no harm to others. -Veracity: To be honest.

Describe at least five (5) roles of the rehabilitation counselor. Describe how you have prepared to fulfill those roles.

1. Vocational counseling: Specialized counseling that's concerned with enhancing career development and securing lasting employment for individuals with disabilities, (Strauser, 2014, p. 179). Includes evaluation, planning, treatment, outcome evaluation; client often is individual served, (Forensic lect 1) Aside from going through an accredited rehabilitation counseling program, I have been counseling clients with different disability backgrounds throughout my practicum and internship experiences. 2. Consultation: process in which one professional seeks the advice of another professional to resolve a specific issue (CRC Code of professional ethics, p. 37 https://crccertification.com/scope-of-practice/); application of scientific principles and procedures in counseling and human development to provide assistance in understanding and solving current or potential problems that the consultee may have in relation to a third party, be it an individual, group, or organization (CRCC Scope of Practice) Includes evaluation and planning; client is often a 3rd party, (Forensic lect 1) I have brought case conceptualization to group and individual supervision sessions; I have contacted other agencies directly about services and accommodations they offer on behalf of client needs (SDS, UI Counseling Services, WRAC, Shelter House, etc.) 3. Assessment and appraisal: selecting, administering, scoring, and interpreting instruments designed to assess an individual's aptitudes, achievement, interests, intelligence, behavior, and disabilities as well as the use of methods and techniques for understanding human behavior in relation to coping with, adapting to, or changing life situations, (Strauser, 2014, p. 179). I have taken an assessment and appraisal class where I practiced and was taught the importance of selecting and using valid and reliable instruments that I am familiar with. During my practicum and internship experiences, I was trained to use and have used assessments with clients. I have worked clients of different backgrounds, interests, and needs requiring different assessments, such as PTSD Checklist (PCL-5), Beck Depression Inventory, etc.) 4. Diagnosis and treatment planning: assessing, analyzing, and providing diagnostic descriptions of mental, emotional, or behavioral conditions or disabilities; exploring possible solutions; and developing and implementing a treatment plan for mental, emotional, and psychosocial adjustment or development. Diagnostic and treatment planning shall not be construed to permit the performance of any act that rehabilitation counselors are not educated and trained to perform, (CRCC scope of practice) Med & Psych Aspects of Disability class; During my practicum and internship experiences, I have administered and interpreted diagnostic assessments, which then led to a discussion about goals and the development of treatment plans. These evaluations and treatments plans have been shared with my colleagues and supervisors for consultation and supervision critique which only enhanced my understanding of treatment planning. 5. Advocacy (environmental, employment, and attitudinal barriers): promoting the well-being of individuals, groups and the profession within systems and organizations. Advocacy seeks fair treatment and full physical and programmatic access for clients, and the removal of any environmental, employment, and attitudinal barriers or obstacles that inhibit access, growth, and development (CRCC Code of Prof... pg. 37 & CRCC scope of practice https://crccertification.com/scope-of-practice/) Working in Pre-ETS to empower students to build skills that will allow them to advocate for themselves.

Counseling theories are based upon certain assumptions about the nature of problems and the human condition. Identify a specific counseling theory that might be useful with a client (or student) who seems depressed. Provide a rationale for your assertion.

A cognitive-behavioral approach to counseling might be useful who seems depressed. This approach focuses on how the way individuals perceive their experiences impacts their emotional state. A part of this approach is Beck's Triad of Depression which provides the primary factors that make depression arise and be maintained. They are: a negative view of self maintained (interpretation of our own self worth), negative filtering of events (constant look at the downside of events), and gloomy vision of the future (have a set of assumptions that things are not going to get better). While looking at Beck's model, a counselor can identify the part of the triad that will be the easiest to influence the most and start from there. With this, changing the "should, must, and ought" statements to be more positive and rational can change the way someone with depression may look at their experiences, both in the moment and any experience moving forward. One important aspect of a cognitive behavioral approach is that the client will begin to learn that most of the time they don't have enough evidence to believe a negative experience will be maintained forever.

Compare two (2) theoretical approaches to individual counseling. Compare and contrast the interventions associated with each theory for short-term counseling.

A cognitive-behavioral approach to individual counseling emphasizes that it isn't the things in our environment or events and situations that we experience that causes stress, anxiety, and other emotional states. What matters is how individuals choose to view those experiences. The way the world is perceived is triggering emotional upsets. Similarly, a choice theory approach emphasizes that the perceptions that individuals have of the world around them influences how their needs' are met and how they have the power to make their own choices to better their lives. Both of these approaches focus on the present/here-and-now. Cognitive-behavioral theory states that people often have these irrational goals they set for themselves that are generally unattainable. This can cause individuals to strive for unrealistic aspects of their life. With choice theory, individuals strive to obtain a "quality world" which is where all of their needs are met and they are achieving. The choices that individuals make in perceiving their environment all effect their behavior and how they continue to see the world. With both approaches, being successful means seeing everything as an opportunity. Each choice made is another opportunity to achieve goals and each experience is an opportunity to change the way we previously thought. Choice theory in itself is already a short-term counseling approach, so its interventions only need to take place over a matter of a handful of meetings. The WDEP System of choice theory stands for wants, needs and perceptions, direction and doing, self-evaluation, and planning. Each of these strategies promote change in the individual through instilling the idea that commitment is their choice and they have the ability to be fully commitment to their development. It allows clients to determine what they want for themselves in a realistic setting and plan to achieve that. Cognitive-behavioral approaches have a similar intervention of identifying their wants in a realistic way, however this intervention turns their previously irrational statements about their current situations into rational statements that allow them to gain a new perspective about their experiences. Cognitive behavioral approaches also use the REBT intervention which stands for rational emotive behavior therapy. This can be used as a brief therapy where individuals are quickly taught how to present and future problems through exercises like role-playing, humor, and homework for relaxation and self-management. Like choice theory, commitment plays a role and is completely up to the individual if they choose to put in any effort.

What is a lifespan approach to career development? What models of career development address lifespan approaches? Describe activities that promote a lifespan approach to career development.

A lifespan approach to career development is the idea that it takes place over one's entire lifespan. As individuals change throughout the course of their life, their career development evolves with them. Super's model addresses a lifespan approach through the development of self-concept that can change over time. Krumboltz's theories addresses a lifespan approach as it states a lifetime of involvement provides learning opportunities that can influence career development as well as that the process never stops so people need to keep their options open throughout life. Activities that promote a lifespan approach to career development are changes in the work environment. Recently, the COVID-19 pandemic changed the game of working from home by making it mandatory, which eventually led some people to never come back to the office. This work environment doesn't work for everyone and can promote change in development across the lifespan. Anything as simple as being born into a family of doctors or a family of athletes influences career development, but later on in life the integration of clubs and independent experiences outside of those initial influences can promote continued development in this lifespan approach.

What forms of interpretive bias are likely to exist in career counseling, and how can career counselors engage in meaningful steps to reduce or eliminate these biases?

A well-meaning career coach with an unconscious racial bias inadvertently steers a student toward career interests stereotypically associated with the student's race. Seeking to hire a technology support specialist, a seasoned career center director eliminates an older candidate from the applicant pool based on unconscious assumptions about the candidate's technology savvy - or lack thereof. A career center manager promotes a team member to a leadership position based on unconscious associations between the individual's gender and their motivation or drive. Implicit bias tests: The IATs measure a variety of unconscious preferences, including age, weight, disability, skin tone, gender, sexuality, and race. Taking an IAT immediately sheds light on where one's bias falls, and often surprises people Once we have identified our biases, the next step is to retrain our brains to think differently - especially about the groups we frequently associate with negative stereotypes.Implicit biases are learned, but they can also be unlearned through a variety of de-biasing techniques. For instance, we are often exposed passively to a barrage of negative stereotypes in the media about specific ethnic groups. Over time, this repetitive exposure can unconsciously affect the way we view ethnically underrepresented individuals, including our own students or team members. By choosing to expose ourselves to positive images of the group - especially counter stereotypes that challenge our unconscious perceptions - we can begin to mitigate the potential negative consequences of our implicit bias.

Describe the differences between an expert and a non-expert witness in court of law in the US. Specifically, describe how a person is determined to be an expert, what an expert can testify to, and relevant court cases that support the vocational counselor as an expert.

An expert witness is a person with extensive experience or knowledge in a specific field or discipline beyond that expected from a layperson. The expert witness's duty is to apply their expertise to give a professional opinion to the tribunal or court on particular matters in dispute. There are two types of witness in the US legal system: a lay witness and expert witness. Unlike a lay witness, who cannot give an opinion about the topics that require special knowledge, an expert witness can testify their opinion within their expertise. An expert opinion must be based on sufficient facts or data and reliable principles or methods. The major function of an expert witness is to express their independent expert opinion based on the information provided. Expert witnesses in vocational rehabilitation can consult, prepare legal documents and testify in cases regarding vocational injury and rehabilitation, including matters such as earnings assessment and capacity, labor market job demand, pre/post injury vocational learning capacity, transferability of skills, career counseling and workplace accommodation, spousal support, the nature of a disability, limitations due to injury/disability, and personal injury. Simply put, A litigator or insurance professional might need to retain a vocational rehabilitation expert witness to support or defend a case involving the issue of employment or an injury. one must be certified by the state and have a license. These experts often have a graduate degree in counseling or psychology plus the appropriate licensure and certifications. This makes them an expert, and it allows them to legally testify on your behalf. This is as opposed to a Vocational Rehabilitation Consultant who cannot testify in a court of law. William MOORE, Petitioner, v. Nancy GRANTHAM, Respondent (1980): This case involves the question whether an expert in vocational rehabilitation can give his opinion concerning a person's future earning capacity, when his opinion is based solely on hearsay information not otherwise introduced into evidence at trial. Findings: "To justify the use of a particular expert's testimony it must appear (1) that the subject is so related to some science, profession or business to be beyond the knowledge of the average laymen, and (2) that the witness offered has such skill, knowledge, or experience in the particular field to make it probable that his opinion will be of assistance to the trier [of fact] in making the factual determination."

List and define the core ethical principles that apply to counseling practice. Describe the process of principle analysis. Illustrate it by constructing and describing a brief ethical dilemma, the ethical choice confronting the counselor, and providing a principal analysis related to this choice.

Autonomy: respect of client's right to make decisions based on their personal values and beliefs Beneficence: always intend to do well and act in the client's best interests Nonmaleficence: do no harm Justice: be fair and impartial to all clients, and be careful not to neglect one client in favor of another; do not let personal bias influence interactions with clients Fidelity: avoid insincere statements and impossible promises; involves concepts of loyalty, faithfulness, and honoring commitments. Veracity: being truthful in one's professional actions with clients and others. Principle analysis: using your intuitive level or moral reasoning; using own values, knowledge, and awareness of these two, along with own preferences in determining how to respond to an ethical dilemma Principle analysis Process: -Intuitive level: your own immediate judgements and actions; INSTINCTIVE approach to taking action - includes ordinary moral sense + facts about situation; "gut feeling" about what to do -Critical-evaluative level: your REASONED judgements and evaluations and actions; REFLECTIVE approach to taking action - includes agency rules, professional codes, laws, principles, ethical theory, moral reasoning; "stop and think about it, then decide" about what to do Brief ethical dilemma: As a rehab counselor, your client shares with you that they have been using drugs Ethical choice -Should you (the counselor) report them? --Intuitive level: Should I tell someone? (Doing drugs is illegal + I'm not the ops + my job is to help this client get a job + this could get in the way of them attaining employment + is this a cry for help?) --Critical-evaluative level: gift example OR I won't report, but I will assess for suicide risk and do a quick assessment of the nature of substance use/abuse to determine if I refer/suggest the client go receive a substance abuse evaluation. (This info could be protected under confidentiality and HIPAA + I can help this individual get a job while utilizing a harm-reduction model + refer them for a substance abuse treatment evaluation to comply with beneficence + I don't want to harm the client by having trusted me with their honesty + I don't want to break fidelity and veracity in promising confidentiality)

Describe the concepts of career readiness and career maturity. How might you assess the readiness of someone to enter a career? What interventions might you use to promote career maturity for a consumer who is having career difficulties?

Career readiness is when an individual can show that they have all the competencies that a general person would need to be prepared for success and life in the world of work. This is having the resources to succeed, but hasn't implemented them yet. Career maturity is when an individual is ready to cope with developmental tasks that are required at each stage of development. This is being able to make a decision or choice that is both realistic and consistent over time, as well as age-appropriate. Career maturity is being able to understand and cope with any obstacles, not just have the resources to do so. Career readiness can be assessed through academic skills taught in school. In order to be ready for a career, one should also be ready for college as many career require a 4 year degree. AP and standardized exam scores can be a good tell of this. Later in college, credentialing and licensure exams can tell career readiness. Career maturity is something that can be improved through hands-on experience outside of an academic setting. Some interventions that can help promote career maturity, but are not limited to, are workshops, job shadowing, internships, and applying for jobs and practicing real interview techniques. Career readiness: Career readiness is the process of preparing students of any age with the essential skills they need to find, acquire, maintain, and grow within a job Career Maturity: the degree to which individuals are prepared to make good educational or vocational decisions. It is usually seen as dependent on their knowledge of themselves and of the world of work, their ability to make decisions, and a positive attitude toward making career decisions. Several competencies associated with career readiness include: Communication skills, Critical thinking skills, Equity and inclusion, Time management, Financial literacy, Leadership, Professionalism, Emotional intelligence, Teamwork, Technology, Stress management. Through Pre-ets/transition activities and interventions, I can document the progress made. Confidence in these areas would show demonstration in career readiness. There are online assessments that can be taken as well. Motivational interviewing, career counseling, interest inventory, counseling on work opportunities.

Decision-making is a complex activity. Describe the stages a person may go through to reach a decision. What interventions might you offer an individual who is having difficulty implementing a decision?

Contemplation (Acknowledging that there is a problem but not yet ready, sure of wanting, or lacks confidence to make a change): Identify the issues Preparation/Determination (Getting ready to change) :Research thoroughly, Identify options/solutions Action/Willpower (Changing behavior): Collaborate with others, Assess feasibility of options, Choose the best option Maintenance (Maintaining the behavior change): Evaluate the results One intervention that a counselor can offer an individual struggling to make a decision is motivational interviewing. In motivational interviewing a counselor will first express empathy to the client about their situation. They will listen, but not tell the client what to do, but also gently persuade them to make their decision. Next they will roll with the resistance by reframing negative statements more positively and acknowledge that ambivalence to making this decision is a normal feeling. Next, the counselor will develop inconsistencies between what the client says and how they behave to help them acknowledge what their values are. Lastly, supporting self-efficacy by exploring past successes will help hold a client responsible and help them carry out their decision. Counselors can also form a working alliance by developing a relationship and setting goals as well as use the INCOME framework.

Discuss two (2) contemporary trends that will expand the role of the counseling profession in the next 10 years. Discuss the evidence for these trends, and the trends likely impact on professionals and consumers. Will these trends likely lead to improvements in consumer care?

Demand for rehabilitation counselors is expected to grow with the increase: in the elderly population (as people are living longer, and older adults are more likely than other age groups to become disabled or injured.). Better health care and hygiene, healthier lifestyles, sufficient food and improved medical care and reduced child mortality mean that we can now expect to live much longer than our ancestors just a few generations ago. Though that's the case, people retire. Those people need help adapting to any new limitations and learning strategies to live independently. Also, there is a need to replace workers who transfer to different occupations or exit the labor force, such as to retire. with the continued rehabilitation needs of other groups, such as veterans who become disabled during their military service and people with disabilities. In 2018, the CDC said that ¼ people have a disability. That statistic alone provides evidence of the ongoing need for resources that only rehab counselors can provide...aka completely individualized services relating to competitive integrated employment. As the field of rehabilitation counseling is becoming more known, the profession is able to serve a greater population of people. A great example of the progress/expansion rehab counseling has experienced is the amendment of WIOA which states that 15% of federal rehab funds must be used to provide certain transition services and supports to improve post-school outcomes of students with disabilities. Pre-ETS are designed to have a positive impact for students with disabilities at an earlier age, leading to better coordination, enhanced communication, and stronger collaboration through the high school transition process, and resulting in increased successful post-school outcomes. I have personally experienced the rapid growth in requests for our services within the school system in Iowa.

The concept of empirically supported therapies and evidence-based practice are crucial to the professional practice of counseling. Briefly define these concepts and some of the ethical reasons that support the adoption of these approaches to counseling. Discuss some major arguments in support of their promotion and the concerns that have been raised in relationship to them.

Empirically supported therapy: aspects of counseling that have been shown to be effective within the literature within certain populations -RTCs here have the highest amount of statistical significance with high internal validities. Evidence-based practices (EBPs): using best available research evidence to support why a treatment works; process of knowing what clinical questions to ask, how to find a best practice, and how to critically appraise evidence for validity and applicability to the particular care situation; typically demonstrated through randomized control trial -best evidence must be applied by a clinician with expertise AND consider the client's unique values and needs -involves the evaluation of the effectiveness of care, and the continual improvements of the process Ethical reasons to use EST/EBP in counseling: -Clients deserve effective and supported treatments -Prevents counselors from just doing whatever they want without having a plan -Provides better protection for counselor in defending/documenting their clinical choices Support for EST/EBP: -research on EBP is based on answering clinical questions about client presenting problems -can evaluate efficacy, efficiency, and effectiveness of treatment with population -usually found through randomized controlled trials - greater statistical support -come from medical field, have stricter research standards Concerns about EST/EBP: -EBP may not be practical -EBP may not work/generalize -EBP may not be feasible

Discuss the relationship between the law and ethics and provide some illustrative examples of how these two (2) dimensions of ethico-legal practice may interface in varying degrees of harmony. Then address the question of how ethical standards indicate the counselor should proceed if there is a conflict between law and ethics.

Ethics are developed by associations to help members practice in a reputable manner. Laws are also based on the same, generally accepted norms, beliefs, customs, and values. However, laws are prescriptive, have been incorporated into code, and carry greater sanctions or penalties for failure to comply Both laws and ethical standards exist to mandate the appropriate behavior of professionals within a particular context to ensure that the best interests of clients are met When laws and ethics seem to be in conflict... counselors will often follow the legal course of action when no harm will come to clients Many ethical standards recognize that other mandates must be followed and suggest that professional counselors work to change mandates that are not in the best interest of their clients. Activity 3.1 provides an opportunity to explore the nuances of an ethical/legal conflict. -Consider a situation in which the law and ACA ethical standards are in conflict with one another (such as when a minor child comes for confidential counseling, but the parent holds the confidence and privilege, or when a minor child comes for substance use or reproductive counseling he/she is entitled to under the law, but he/she is engaging in dangerous behaviors in these regards). Role-play with a peer how you might approach your client in explaining the conflict and your ethical standards. What might you do to begin working toward changing legal mandates that are not in the best interest of your client? (Erford, 2018, p. 71) Tarvydas' Integrative Ethical Decision-Making Model Stage 1: interpret the situation through awareness and fact finding Stage 2: formulate an ethical decision Stage 3: weigh competing factors and select an action Stage 4: plan and execute the course of action -When there is conflict, common sense should prevail -If there's a therapeutically logical reason to follow one particular mandate over another, one should follow the logical course of action and document what one does and why. -If a policy/guideline/regulation is not in the client's best interests then ethically you should work to change the mandate. -Follow the mandates that apply to your work setting. -Ethics stay with you but the law changes everywhere you go. -If two laws are competing, go with whatever is more restrictive -Professional counselors need to make their clients aware of any conflicts with their ethical standards (ex: mandatory reporting) --Ex: if you're working with a suicidal teenage client but believe telling the client's parents would perpetuate an abusive situation, you should handle the situation as an abuse case and inform child protective services about the suicidal behavior

Describe how you would use microcounseling skills to respond to someone you thought might engage in self-harm. Be specific in describing the skills, questions, and the actions you would take to assist the client/consumer/student.

First, it is inappropriate to make judgements. Use a calm voice, do not panic, and never make assumptions. I would assure them that I am only here to help and support him, and that they are in a safe space. If I saw an injury that might be self- harm, I would ask about it in a way that lacks confrontation or challenge. I wouldn't pressure them to speak though. Ask them what will help them to feel safe? Ask if they have thought about ending their own life/seriously hurting themselves. Asking gives them a chance to talk about it. I would also ask if they have any resources if they were ever in a situation where they were worried about it escalating. Self-harm is not an emergency, but still needs to be taken seriously. Encourage and support them in seeking professional help by someone qualified. When SI is disclosed, I should explain techniques and interventions that will be used specifically to address SI. I should also be very clear about the duty to protect and how SI might lead to mandated reporting, such as if the client develops suicidal intentions or if SI results in a major health risk (e.g., large, infected wounds). Finally, I will talk to my supervisor.

What is the economic impact of disability on: 1) society and 2) the individual?

For society, the economic impact of disability is seen in poverty rates with individuals with disabilities being significantly higher than individuals without. Also, just in Iowa, around 43% of households receiving food stamps have one or more people with a disability living in the home. The low wages that seemingly have an effect on the individual ends up impacting society as the amount of households with food stamps is significantly high for individuals with disabilities, and unemployment rate for individuals with disabilities is higher as well. The economic impact of disability on the individual is shown in many ways. Individuals with disabilities receive significantly less pay than their non-disabled colleagues. As educational attainment increases, so does the gap between pay for individuals with and without a disability.

Describe the ethical use of assessment tools with consumers. Describe how you would ensure that an assessment tool was culturally sensitive to a consumer from a diverse background.

From the moment we first meet with a client through the last session had together, counselors are always assessing and gathering data --The more info we have about a client, the better we are able to help the client achieve successful outcomes --Generally speaking, assessment serves 4 primary purposes in the counseling process: ----Diagnosing ------------Conceptualizing client probs ----Screening ------------Assessing client probs ----Treatment planning and goal identification ------------Selecting and implementing effective treatment ----Program evaluations ------------Evaluating counseling effectiveness

How can counselors act as advocates/change agents in addressing systems of oppression that influence client access and outcomes in the counseling process?

Helper - provide services and skills designed to help those without privilege have increased decision-making power over their lives and understand the systemic oppressions influencing their lives. A helper might be someone who volunteers at the front desk of an LGBTQ+ Advocate - engages in legal methods of social change (e.g., legislative policy, legal challenges to unjust law and unjust treatment of individuals and communities), builds coalitions to work toward explicitly stated demands for justice, and assess how these justice demands come to fruition or get stalled. An advocate might be someone who works at a local law or policy center or think-tank seeking to influence and shift racially unjust policies Organizer - strategically focuses on long-term demands for justice and uses grassroots strategies, leadership development, consensus building, and coalition building to develop specific routes toward justice. An organizer might be someone working to improve relations between people of color and the police in a specific neighborhood over a long period of time. Rebel - uses protest and street activism as vehicles for social justice change, uses specific measures of nonviolence to challenge people with power and unjust institutions, and focuses on strategically addressing specific issues and injustices in a public manner with a plan for success. A rebel might be a White person who is willing to say the hard things in a racial justice group that people of color feel they can't say or won't be taken seriously if they do

The contemporary history of rehabilitation counseling in the United States began in the late 1800s. Describe the major legislation that influenced the professional and major legislation that led to the inclusion of diverse consumer groups as rehabilitation consumers. How has the legislation impacted the types and numbers of consumers counselors serve?

Historically, the inception of rehabilitation counseling as a profession occurred with the Smith-Fess Act of 1920. This act, also known as the Civilian Vocational Rehabilitation Act because it was based on the Soldier's Rehabilitation Act, provided 50% matching funds to states that developed state plans for assisting individuals with disabilities with counseling, education and training, prostheses, and job placement services. The Smith-Fess Act provided funding only for those with physical, not psychological/psychiatric, disabilities, and excluded the blind, but eventually led to expansion of the rights of individuals with disabilities and increased the need for rehabilitation counselors. The American Rehabilitation Counseling Association (ARCA) was established in 1957 and is a division of the American Counseling Association. ARCA's mission is "to enhance the development of people with disabilities throughout their life span and to promote excellence in the rehabilitation counseling profession." The Rehabilitation Act of 1973 eventually called for the formal establishment of civil rights for people with disabilities, prohibiting discrimination on the basis of disability in any federally-funded program (including schools, employment, and federal contractors), establishing a federal board to ensure accessible architecture and public transportation, and stressing consumer involvement in the rehabilitation process. The Workforce Investment Act (WIA, 1998) then followed the ADA, and promoted the idea that individuals with disabilities should be "served in an inclusive and integral way" (Elliott & Leung, 2005, p. 325). Most recently, the Workforce Innovation and Opportunity Act (WIOA, 2014) was signed into law with additional stipulations seeking to encourage competitive integrated employment for workers with disabilities, another attempt at improving societal representation, equity, and inclusion.

Consider the advantages and disadvantages of two (2) distinct career development theories/approaches that can be used to assist clients/students who are reporting a strong gender bias toward their career options. Identify the strengths and limitations of each approach in this context.

Holland (social contextual) RIASEC: a theory of personality that focuses on career and vocational choice. If a student is picking careers based solely on gender bias the holland code will most likely be able to pick this up and allow for greater discussion on the bias Gottfredson: Children learn their orientation to gender roles/sex roles at an early age, 6-8. Counselors can do their part in creating awareness of different gender careers starting early on in elementary school. At the older levels the counselor can bring awareness to this bias. -Invest in critical thinking and learning, thinking about messages that they hear all the time. - Represent cognitive diversity - optimize experience: non stereotypical workers, part time jobs, community service, field trips. Self insight is very important in gaining a complex idea of career interests, life goals, group discussion, gender stereotypes, etc. -Counselors can provide support and information to help these students gain awareness.

You are working in a small rural setting with very little diversity but a very large and wide range of needs. You have your hands full each day with different issues that arise. Recently, you have become aware of a very large need for some diversity/multiculturalism/social justice awareness. Provide three recommendations on how you would increase multicultural awareness within your setting.

In an office setting, you could interview a diverse group of your coworkers, or even clients and share their stories amongst your office Consider Launching employee resource groups- they give underrepresented employees a voice and can help to encourage collaboration and allyship within your organization Ask questions, broach. Acknowledge differences in race, sex, culture and listen to understand each individual.

Critically evaluate one (1) of the career development models from a cultural perspective. How does the model address diverse cultural expectations regarding career and the world of work? What worldview does the model pre-suppose? How can you use this model in a culturally sensitive manner?

Holland's Theory follows a person-environment mindset, where their personality determines how well fit they are to a certain type of career path. Job satisfaction is greatly improved when congruence exists between personality and job environment which can be examined within one's culture. This model addresses how one's upbringing and parents can affect preferences in activities, interests, and values, which then affects one's tendency towards certain personality types. This model does a poor job of addressing diverse cultural expectations in the sense of gender. Because of the simplicity of the model, women tend to be scored in fields of work that are highly female dominated, like lower level teaching, nursing, and secretaring. Through a worldview, Holland's theory presupposes that personality and job environment are correlated with job satisfaction. High congruence among the two should predict high job satisfaction. You can use this model in a culturally sensitive manner by considering what occupations are most commonly worked by individuals of different cultural backgrounds. Just because a female is assessed using this model and their personality correlated to highly female dominated jobs, then look outside the box. Social type is more female dominated, but there are many other jobs that can be considered. Step outside the common worldview and see what else is out there.

Barb is a Hispanic, 30-year-old woman who does not speak fluent English. She has a master's degree in accounting but is having trouble finding a job. She is very discouraged because she took an Interest Inventory at a Community College Career Center which indicated she had personality traits like persons who were employed in service industries. She has come to you for assistance and to build her confidence. What would you tell Barb about the assessment results? What model would you use to guide your intervention?

Interest inventories can be a great starting point in your search for a career that you will love. Interest inventories look at a person's likes and dislikes, their favorite activities, and their personality. Then they compare those qualities with the qualities of people who are already working in specific careers. Your results may show that you have a lot in common with specific professions, such as in the service industry. Knowing this information can help you to take the next step in researching different careers. But keep in mind that an interest inventory isn't the final step in your career exploration! Even the best inventories are only making suggestions that you will have to research further. Just like a puzzle, there are many pieces that need to go together. An interest inventory is just one piece of a large puzzle. Sometimes, people think an interest inventory is solving the puzzle for them by choosing their career. But you have to finish the rest of the puzzle! I would use motivational interviewing to find out more. The four steps of the MI process are engage, focus, evoke, and plan. Compassion and empathy are foundational in MI. First, I would engage Barb using open-ended questions, empathic reflections, autonomy-supportive affirmations, and rolling with resistance. This is to build rapport, gain a deeper sense of understanding, and learn more about the JC' background. Second, Focus on understanding what Barb is saying and how he or she feels. Empathize with her perspective. Focus by reflecting, summarizing, and developing discrepancies. To be an effective guide, you need to know where you're going. Focus on why she doesn't like the service industry. Focus on the highlighted skills in the inventory to discover something new. The next step is to respectfully evoke your client's thoughts, feelings, motivation, and concerns. That is, get the discussion going and directed at underlying concerns that might increase or decrease motivation in certain job areas. The last step is to talk about a plan. Might be another interest inventory, job shadows, informational interviews, etc. In some cases, the plan may just be to talk about it again later if the person isn't ready.

Discuss how you will consider cultural norms when using microcounseling skills.

It is necessary to recognize that valuing and respecting cultural differences requires a commitment to life-long learning and being able to make sound ethical decisions within diverse cultural contexts.This includes how I utilize microcounseling skills and how they are interpreted. Common factors that vary across cultures include: communication patterns, values, gender roles, emotional expressions, mental health stigma, presentations of distress, counseling expectations, body language, and behavioral norms. I must interpret client presentations from a broad cultural perspective instead of using only my own cultural groups or previous client experiences as reference points. I can actively demonstrate commitment to cultural competence by behaviors that reflect attitudes of respect, acceptance, sensitivity, commitment to equality, openness, humility, and flexibility. I must invest the time to know clients and their cultures, not only through ongoing client interactions, but also through the use of outside resources, cultural training seminars and programs, cultural events, professional consultations, cultural guides, and clinical supervision.I can also integrate culturally relevant information and culturally relevant themes to more fully understand clients and identify their cultural strengths and challenges. Finally, I can ensure to select culturally appropriate tools and interventions to ensure success.

Define microaggressions. How can the experience of microaggressions in the counseling process influence client retention and subsequent outcomes?

Microagressions are subtle, indirect, and often unconscious acts of discrimination against a marginalized group. The marginalized groups are usually a racial or ethnic minority, but can be towards any minority group like individuals with disabilities. Experiencing microaggressions in the counseling setting can lead to clients being unwilling to open up, but also can create an unsafe environment for the client. Instead of making progress in a setting that is supposed to instill growth and safety, the client may not only be resistant to counseling with this counselor, but may be resistant to any counseling in the future. Experiencing microaggressions can cause psychological distress, symptoms associated with trauma, and an overall reduced mental well-being. Counseling should be a safe space where clients can improve these aspects of their lives, not continue to dwell in them.

Are standardized assessments a valid and reliable method of evaluating persons with disabilities? Describe how you would determine if an instrument was valid and reliable for a given population.

No. Barriers to success on standardized exams for students with disabilities include lack of access to learning the material on the tests. This sometimes results from "restrictive" placements, in which children with disabilities are separated from other students, then not given full access to the mainstream curriculum. Students often do not receive services during test time because teachers are supervising the testing. Other barriers to success include lack of access to accommodations that are sometimes helpful. These include extra time, text-to-speech tools and Braille. Another problem is the overreliance on standardized test scores to make important decisions about student placement, advancement from grade to grade and graduation. The harms of these policies tend to be greatest for this student subgroup, including an extreme narrowing of curriculum to little more than test preparation. SWDs and English Language Learners are the groups most likely to be denied high school diplomas as a result of failing high school graduation tests. Many commonly used test accommodations, such as extra time, can be a double-edged sword. Extra time may help some SWDs do better. Often, however, it leads to such students spending many hours working to complete a flawed standardized exam instead of focusing on their individual learning goals, without any score gain. When students with disabilities are subject to high-stakes decisions based on standardized test results, they are at higher risk of negative consequences such as repeating a grade, dropping out, or completing school with something other than a full-fledged high school diploma, such as a certificate of completion. The latter has little value in terms of the opportunities to pursue higher education or obtain employment. Multiple measures of student knowledge and skills are the best, most fair and accurate approach to assessment for all students, especially for students with disabilities. The priority should be on high-quality, teacher-led classroom assessments that provide multiple ways and opportunities for students to demonstrate their knowledge and skills. --Test retest reliability/ alternate form reliability --Content validity/construct validity/ criterion-related validity

One common assumption associated with counseling theories is the notion that emotional problems can be improved via interventions and participation in a counseling relationship. Describe the factors that you think are the most essential components and features of interventions and relationships that bring about emotional change.

One factor that is believe is essential in bringing about emotional change is congruence in a therapeutic alliance. A counselor can't expect their client to be real and genuine in sessions if the counselor can't be genuine back. A client reaches out to their counselor for support and we as counselors model the idea of being a true human being. This type of self-disclosure should be used sparingly and not taking priority in the session, but an appropriate time and amount of self-disclosure can allow the client to open up and connect emotionally where they may not have been able to before. Another factor that I believe is an essential component of interventions is learning new ways of thinking. Many interventions of approaches focus on changes in thinking patterns about experiences. Adapting the framework of the way someone perceives an experience can bring about emotional change as they can begin to view things more positively, or even negatively. Some people need to be able to alter their perceptions in order to grow emotionally. One last factor that I believe is an essential component to bringing about emotional change is showing empathy to the client. We have been taught many times that showing empathy is a greater strength to have as a counselor than providing sympathy. Empathy shows that a counselor not only has formed that connection with the client, but they also can share that experience with them without trying to assume how they feel. This can promote emotional change in a client because they no longer have to keep everything to themselves and have someone they are able to share their journey with.

Individual theories of counseling include certain cultural biases. Compare and contrast the cultural contexts of two (2) theories of individual change. Describe cultural biases of the theories and how the proponents of those theories address cultural concerns. Discuss methods that you may use to make individual change theories more relevant to persons from diverse cultures.

PC Multicultural Contribution: -Focus is on breaking cultural barriers and facilitating open dialogue among diverse cultural populations Main strengths: -Respect of clients' values -Active listening -Welcoming differences -Nonjudgmental attitude -Understanding -Willingness to allow clients to determine what will be explored in sessions -Prizing cultural pluralism CBT Multicultural Contributions: -Focus on a collaborative approach that offers clients opportunities to express their areas of concern -The psychoeducational dimensions are often useful in exploring cultural conflicts and teaching new behavior -Emphasis on thinking (as opposed to identifying and expressing feelings) is likely to be acceptable to many clients -The focus on teaching and learning tends to avoid the stigma of mental illness -Clients are likely to value the active and directive stance of the therapist. PC Multicultural Limitations: -Some of the core values of this approach may not be congruent with the client's culture. -Lack of counselor direction and structure are unacceptable for clients who are seeking help and immediate answers from a knowledgeable professional CBT Multicultural Limitations: -Before too quickly attempting to change the beliefs and actions of clients, it's essential for the therapist to understand and respect their world. -Some clients may have serious reservations about questioning their basic cultural values and beliefs. -Clients could become dependent on the therapist choosing appropriate ways to solve problems. Methods: pg 303 in Microskills Book: -Establish a strong therapeutic relationship and let the client know that I respect them. -Adapt my treatment to their specific ethno-cultural identities (more/less structured, homework, appropriate skill development) -Regularly check in on how the treatment is meeting the client's needs and expectations. React to client feedback asap and adjust the treatment accordingly. -Use methods and techniques that are appropriate and acceptable within the client's culture.

Explain the difference between personality and personality disorder. Then, for personality disorder: a) describe the three (3) P's of personality disorders; b) name and describe the clusters of personality disorders; and c) list the specific disorders in each cluster and its core feature(s).

Personality is people's traits, coping styles, and ways of interacting with others around them. They are adaptable and can bend to the expectations and norms of society. Personality disorders are not adaptable and an individual's behavior remains constant across different situations and societal interactions. The first P of personality disorders is problematic, which means that present problems are caused by difficult ways of relating to society. The second P is persistent, which means that the behavior continues regardless of obstacles or efforts to stop. The last P is pervasive, which means that the behavior is extensive, ongoing, and long-lasting. Cluster A is odd and eccentric behavior. Cluster A includes paranoid, schizotypal, and schizoid personality disorders. Paranoid personality disorder is a suspicious, general mistrust of others. Schizotypal personality disorder consists of interpersonal deficits and perceptual disturbances. Schizoid personality disorder is the inability and lack of desire to form social relationships. Cluster B is dramatic, emotional, and erratic behavior. Cluster B includes antisocial, borderline, histrionic, and narcissistic personality disorders. Antisocial personality disorder consists of blatant disregard and violation of the rights of others. Borderline personality disorder consists of unstable relationships, poor self-image, and a lack of emotion regulation. Histrionic personality disorder consists of attention seeking and self-dramatization. Narcissistic personality disorders include excessive admiration, and overvaluation of others' opinions and reactions. Cluster C is anxious and fearful behavior. Cluster C includes avoidant, dependent, and obsessive compulsive personality disorders. Avoidant personality disorder includes feelings of inferiority and inhibition. Dependent personality disorders include enmeshment and external guidance and support of others to achieve desires. Obsessive compulsive disorders include inflexible behaviors, perfectionism, and value of order and rules.

Define psychopathology. Identify three (3) types/groups of psychiatric disorders in the DSM-5. What are the core features of each of the types/groups of psychiatric disorders?

Psychopathology is the study of significant causes and processes in the development of mental illness. In a broader scope, mental illness can include mental disorders, emotional problems and distress, and maladaptive behaviors. Three groups of psychiatric disorders are anxiety disorders, personality disorders, and mood disorders. broadly, its the study of mental disorders. Features of anxiety become classified as anxiety disorders when vague feelings of apprehension and uneasiness are combined with automatic physical experiences, like tightness in the chest, muscle pain, dizziness, and palpitations, that make the individual think they are experiencing a medical medication condition that isn't related to anxiety, when really they are in a non threatening situation. These emotional and physical experiences also have to be recurrent and increase in intensity that it begins to affect the individual's ability to function. To qualify with a personality disorder an individual's behavior must be pervasive, inflexible, and stable over a long period of time. It also must cause significant distress or impairment in functioning in at least two areas of cognition, affectivity, interpersonal functioning, or impulse control. The individual's behavior doesn't adapt to new experiences, but remains consistent across all situations. Individual's don't usually see their behavior as problematic. The symptoms of mood disorders are classified using the term episodes. Qualifying for a mood disorder consists of an individual experiencing a specific episode related to the specific mood disorder (i.e. bipolar I, bipolar II, major depressive disorder, etc). The different episodes are manic (elevated mood), hypomanic (elevated mood, lower level), and depressive episode (decreased mood). Neurodevelopmental disorders: start in the developmental years, include executive functions, social impairments, and intelligence. Depressive: Prevalence of sad, empty, or irritable mood; somatic and cognitive changes impact the individual's ability to function.

Compare and contrast the practice of rehabilitation counseling in the private sector with the practice of rehabilitation counseling in the public sector. What are similarities and differences in the practice of rehabilitation counseling in the private vs public sector?

Public rehab: generally state-run, meaning they are funded by government programs and taxpayer dollars.Public sector rehabilitation counselors assist clients to minimize their barriers to employment and maximize their opportunities. Barriers to employment are decreased through services that address psychological, vocational, social and behavioral issues (O'Sullivan & Bates, 2014). Additionally, services include, but are not limited to, psychological and vocational assessments, individual and group counseling, training, job placement, and assistive technology. Public sector rehabilitation counselors are known to work with more severe disabilities and clients. Private rehab: Private sector rehabilitation counselors provide services that include case management, forensic evaluations and testimony, life care planning, and disability management. Private sector rehabilitation counselors work in case management in various settings such as major business corporations, health insurance companies, and attorneys' offices. Rehabilitation counselors who work within forensics help provide expert testimony in litigation regarding workers' compensation or personal injury cases. Providing expert testimony requires private sector rehabilitation counselors to have knowledge of the law. Those who provide disability management services work with injured workers to help them throughout their recovery. They facilitate treatment to help with the return to work process and to reduce the length of time an individual is off work. Other Differences: Rehabilitation counselors in the private and public sectors practice in different environments (with varied laws, rehabilitation goals, duration of services), and minimal consideration has been given to the diversity of ethical dilemmas that these practitioners encounter in their professional roles.

Describe the multicultural and social justice counseling competency framework. How is this framework different from the predecessor conceptualization of multicultural counseling competency?

Quadrants illustrate intersection of identities and the dynamics of power, privilege, and oppression that influence the counseling relationship. Developmental domains reflect the different layers that lead to MSJ-competence: 1. Counselor self-awareness 2. Client worldview 3. Counseling relationship 4. Counseling and advocacy interventions Embedded in the first 3 domains are the aspirational competencies: attitudes and beliefs, knowledge, skills, and action (AKSA). The socioecological model is incorporated within the counseling and advocacy interventions domain to provide counselors a multilevel framework for individual counseling and social justice advocacy. Multicultural and Social Justice Counseling Competencies figure: It differs from its predecessor in that the MSJCC includes intersectionality (ex. African American gay man); represents individuals as part of a larger ecosystem; intersects privileged and marginalized status

Because of the participation of human subjects as research participants, counselors who conduct research should take into consideration ways in which to protect these individuals' welfare. Describe the role of ethics, as supported by CRCC (RMHC students), AMHCA (CMHC students), or ASCA (SC students) and ACA (ALL students), in the research process with attention to both the researcher and the research participant.

RCs who conduct research are encouraged to contribute to the knowledge base of the profession ---They promote the welfare of individuals with disabilities as well as a clearer understanding of the conditions that lead to a healthy and more just society ---RCs minimize bias and respect diversity in designing and implementing research ---RCs understand the need for research that includes diverse populations, including individuals with disabilities and other under-represented groups ---Standard I.1.b. Use of Human Subjects - RCs plan, design, conduct, and report research in a manner that is consistent with pertinent ethical principles, applicable law, host institutional regulations, and organizational and scientific standards governing research with human subjects. They seek consultation when appropriate. ---Standard I.1.e. Independent Researchers - when RCs conduct independent research and do not have access to an IRB, they are bound to the same ethical principles and laws pertaining to the review of their plan, design, conduct, and reporting of research. Independent researchers not familiar with IRB standards seeks appropriate consultation. ---Standard I.2. Rights of Research Participants ---------Standard I.2.a. Informed Consent in Research - individuals have the right to consent to or decline requests to become research participants. RCs obtain consent from participants prior to initiating research. In seeking consent, RCs: ---------------Accurately explain the purpose and procedure to be followed ---------------Identify any procedures that are experimental or relatively untried ---------------Describe any attendant discomforts/risks ---------------Describe any benefits/changes in individuals/organizations that might be reasonably expected ---------------Disclose appropriate alternative procedures that would be advantageous for participants ---------------Offer to answer any inquiries concerning the procedures ---------------Describe any limitations on confidentiality ---------------Describe formats and potential target audiences for the dissemination of research findings ---------------Instruct participants that they are free to withdraw their consent and to discontinue participation in the project at any time without penalty.

Provide your argument for the need of rehabilitation counselors to be trained in medical aspects of disability. What is the role of standards of practice and professional ethics in this training process?

Rehabilitation counselors need to be trained in medical aspects because they don't just need to be able to help individuals with disability work towards employment or the ability to live independently, but they also need to be able to advocate for them as well. It is hard to advocate for a client when you don't fully understand the medical side of their disability. Each disability is completely different from another, and it varies for each individual person. Knowing the functional limitations of a disability can help a rehabilitation counselor advocate for a client while moving forward with their services. Some limitations can become more prominent with time because of how a disability progresses. Without training in the medical aspects of a disability, rehab counselors would not know this information and assess a client incorrectly or help place them in a job that could worsen their work capabilities. Role of standards of practice: Role of professional ethics in the training process:

Describe reliability and validity. Why are these concepts important in the ethical use of assessment devices? How does one determine if an assessment is reliable and valid? Make sure you discuss specific methods of determining reliability and validity.

Reliability: consistency, precision; degree to which SCORES on an instrument are measured consistently (the scores not the instrument); ratio of true score variance to total variance (true score + error variance) --Test scores are trustworthy to the degree that they accurately assess some trait of ability by minimizing randomly occurring testing errors. --Unreliability shrinks the observed effect size through attenuation (lecture 6) Validity: accuracy; usefulness; degree to which test scores measure what the test claims to measure, and the degree to which it measures it accurately. --Verifies exactly what the test is measuring and the accuracy with which that measurement occurred. --For an assessment to be proven valid, it must first be proven reliable. Ethical Importance --"Professional counselors should choose assessments that are the most appropriate for the targeted purpose of the assessment and for the clients they are assessing. The implication of "fit" for clients from diverse populations is particularly important. Professional counselors must explore each instrument's psychometric properties and ensure its appropriateness and usefulness for clients from diverse cultures," E.7 Conditions of Assessment Administration, Assessment book, ch. 1) --Instruments with high reliability and validity scores will help you to identify differences among groups if those differences truly exist. If a test has higher reliability in the scores it produces, then a counselor would know the results are likely to be possibly (as long as the instrument had the appropriate reliability for the client/population it was being used for). Determining the reliability and validity of an assessment: Reliability: Classical test theory: Standard error of measurement:

Describe one (1) theory of individual change. Identify the microcounseling skills congruent with the foundational theory of counseling that you have described. Identify any microcounseling skills that would not be congruent with this approach.

Rogerian theory assumes that all humans are innately good and creative. Because of this, humans have the ability to become very goal-directed and can learn to grow naturally within the presence of support. People have the potential for understanding themselves and their own problems without direct intervention, but self-growth comes from a therapeutic relationship. In order for this approach to be effective, there must be a positive working relationship between the client and the counselor. This is considered to be a person-centered approach to counseling. The first thing the counselor needs to have is congruence and genuineness about who they really are and share that with the client. They also need to show empathy towards the clients feelings and beliefs as if they also have the same feelings. Lastly, unconditional positive regard must be given in a non-judgmental way in an attempt to not disrespect the client's experiences. This alone can instill change in an individual as they begin to learn more about their own capabilities. The nonjudgmental listening cycle as a whole are microcounseling skills that are congruent with this approach. Specifically, reflecting meaning and feeling are two skills that would be useful to this approach. While still being open with the client, the counselor can express what they are picking up on in the conversation. Through this the counselor can express empathy about the clients feelings. Paraphrasing and summarizing are useful in this approach as well because the counselor can't express empathy and provide unconditional positive regard if they are not able to fully capture what the client is saying. These check ins can make sure everyone is on the same page. Goal setting would be integral here as the person-centered approach emphasizes that individuals have the potential to achieve their goals. One microcounseling skills that would not be congruent with this is psychoeducation as the pair should be seen as equals, not one superior to the other. Changing irrational beliefs is also not very congruent with this approach as the core idea is that humans are trustworthy individuals and their feelings and statements need to be validated. Challenging them may be seen as invalidation.

Individual theories of counseling must be applied in an ethical manner. Describe a theory of individual change. Describe the interventions that correspond to that theory. Describe an ethical dilemma that may arise from an implementation of the theory. How would you resolve this dilemma within the framework of the theory you have described?

Rogerian theory assumes that all humans are innately good and creative. Because of this, humans have the ability to become very goal-directed and can learn to grow naturally within the presence of support. People have the potential for understanding themselves and their own problems without direct intervention, but self-growth comes from a therapeutic relationship. In order for this approach to be effective, there must be a positive working relationship between the client and the counselor. This is considered to be a person-centered approach to counseling. The first thing the counsleor needs to have is congruence and genuineness about who they really are and share that with the client. They also need to show empathy towards the clients feelings and beliefs as if they also have the same feelings. Lastly, unconditional positive regard must be given in a non-judgmental way in attempt to not disrespect the client's experiences. This alone can instill change in an individual as they begin to learn more about their own capabilities. Motivational interviewing is another intervention that promotes individual change. This is where the individual goes through the stages of change: precontemplation, contemplation, preparation, action, and maintenance. Stages don't have to be achieved linearly as fluctuation often occurs. However, support is given by the counselor that matches each stage. For example, in the contemplation stage the counselor may point out that there is a disconnect between the clients thoughts and behaviors. One ethical dilemma that could arise with a Rogerian, person-centered approach is that because a very personal therapeutic alliance must exist between counselor and client, the client may be confused on where the boundaries lie when it comes to their relationship outside of the office. While still using a person-centered approach, I would continue to be open an honest with my client about the intentions of the arrangement that we are in. It is important to remain consistent with the client throughout this process. I would also continue to validate their feelings and let them know that they are heard. I wouldn't want to disrespect the experiences that the client has been having, but as long as I continue to be open with the client about myself, I will be able to discuss our boundaries and make sure they are newly set in a clear way.

Your cousin John is 25 years old and has worked full-time since he was 18. He hates his job. Last week, he broke his leg while hunting and will be off work for 3 weeks. He talked to his neighbor who suggested he just quit work and apply for SSDI. John knows you learned a lot about that "free government money" while in graduate school and had called you up for advice. Please tell John about the following: 1) what SSDI is; 2) who is eligible for SSDI; 3) what benefits John could expect to get, if approved; 4) how long John could expect to get benefits, if approved; 5) how one would apply for SSDI; and 6) what happens if John gets another job (e.g., can he keep his benefits?). Should John quit his job in order to increase his chance to successfully obtain SSDI benefits?

SSDI stands for Social Security Disability Insurance. The government determines eligibility, however you must be younger than the retirement age, there must be medical documentation of a disability that meets the standards of Social securities definition of disability and be unable to work because you have a medical condition that is expected to last at least one year or result in death. John would not be approved, as his disability is short term, and when it heals, he will be fully capable of returning to work. he would keep them until he either receives a job that is deemed "substantial gainful activity" or when he reaches retirement age. You can apply for SSDI online or make an appointment to go in person. If John gets another full time job making enough money, SSDI will determine that you don't meet their definition of disability anymore. John should absolutely not quit his job, as the job is not specifically what determines eligibility for SSDI.

In your own words discuss at least four (4) unique ethical considerations that must be taken into account when forming a group? And how are these considerations impacted when working with vulnerable populations?

Screening --It would be unethical to skip the screening process, as it is a way of ensuring an admitted member can benefit and contribute to the group --Impact: this would violate beneficence and nonmaleficence by not protecting the group members from harm Personal relationships between group leader and member(s) (favoritism) --Avoid outside contact with group members as it could lead to favoritism or a distraction from important personal topics within the context of the group --Impact: this would violate justice as not everyone's well-being would be promoted; this could develop distrust Dual relationships --Avoid admitting friends, family, or colleagues to your group as this carries the potential for harm because of the inherent vulnerability of the client and the imbalance of power between the counselor and client -------This would violate nonmaleficence as it could cause harm through an imbalance of power; this could develop mistrust and stunt a client's autonomy and healing Closings and follow-ups --When closing a group, leaders should make themselves available to group members for consultation as well as follow up meetings after group has ended. This is to ensure the members are able to adequately assess the impact the group has had on them and are unlikely to continue working on their goals as specifically as they would otherwise ------This would violate justice and nonmaleficence as it has the potential for abandonment and harm. This could be a trigger for trauma or traumatizing.

Describe three (3) of the techniques or skills that are a part of microcounseling. Describe how these skills foster a relationship between counselor and client/consumer/student. Describe any cultural and an ethical consideration you may have to address prior to using these skills with a consumer or student.

Skill one: reflecting on non verbal behaviors. Understanding that non verbal behaviors and meanings may vary across cultures and/or be used in a different way then my culture might. Being aware of that and not jumping to conclusions is very necessary. Skill two: building rapport. Rapport establishes trust between you and your client. The more you get to know your client, and the more they get to know you, the easier the process and the more comfortable they are. When working across cultures, it's important to be aware of my own biases. It's important to be open and ask questions if I don't know something, instead of pretending I do. Ensuring the client is in a safe, comfortable space is number 1. Also, staying professional while building a strong therapeutic alliance. Skill three: Feedback. Providing different opportunities for clients to provide feedback on their counseling sessions/outcomes. It will help learn more about areas the client wants to focus/ doesn't want to focus, etc. It allows the client to be heard and ensure that you are utilizing time as efficiently as possible. Some clients may not be comfortable putting their true experience, so providing opportunity for anonymity is important. As well as creating a format that is non-discriminatory or exclusive to any populations.

Debates continue in the career development literature as to whether problems with career choices are primarily related to cognitive, developmental, or social/contextual factors. Which of these three (3) sets of theoretical assumptions best explain why an individual might be unwilling to consider a particular career path despite clear evidence that he or she actually has tremendous aptitude, potential, and skill for this particular career option?

Social/contextual factors --Individual's values must also correspond with that role --------Values = largely shaped through a social/contextual factor lens. Putting aside cognitive and developmental factors, all of the career models look (to some extent) at interest and values as one of the predicting factors for satisfaction (and consideration of) a particular career option Developmental assumption: Gottfredson talks about how gender influences the way people choose careers. Circumscription: eliminating career options. Kids might think they want to be something but then later eliminate it due to conflict between self concept. cognitive growth (age related growth), self creation (self directed development), circumscription(elimination of least favorable occupations - what is acceptable or unacceptable for a person like me?), and compromise (accommodate constraints on career choice). Stage 1: orientation to size and power(3-5) Stage 2: Orientation to sex roles (6-8) Stage 3: Orientation to social valuation (9-13) Stage 4: Orientation to unique self (14-on) Someone might get stuck early on. Not have enough role model

Describe and contrast a standardized and non-standardized method of assessing cognitive functioning. What cultural consideration should you reflect upon prior to evaluating a person from a diverse culture using a standardized intelligence test?

Standardizing Testing: refers to the formal process in which a specific set of rules and guidelines related to administration, scoring, and interpretation are followed consistently to ensure accurate results over a period of time and across populations --------Ex: instruments such as achievement tests, aptitude tests, and personality tests fit this description ---------Validity and reliability are known Non-Standardized Testing: refers to a process of gathering info without adherence to a strict set of rules or guidelines -------Ex: may include clinical interviews - e m ven when such interview follow a formula or pattern, deviations in adinistration occur because of the personal nature of the interactions and of addressing the client's personal needs. -------Such assessments may not adhere to a rigid administration, scoring, and interpretation like that of standardized testing When working with diverse populations: the culturally competent counselor's best course of action is to select tests and assessments that were specifically designed for the groups with whom he or she is working with. -------unfortunately, this is not an option that is always available. Many existing tests were created using homogenous normative samples that were not very diverse. -------As counselor's evaluate tests, they must ensure they are making note of the characteristics and demographics of the norm group and recognize any potential limitations to using the test that might arise when working with other populations. -------Failure to do so may result in the counselor having invalid test results that really do not tell them anything useful about the client that can be used in the counseling process.

Bob owns Big Bob's Warehouse and he wants to hire a Certified vocational counselor to help him with applicants who have a disability. His lawyer, Shad E. Sam suggested to Bob that he hire a vocational professional to redesign all warehouse jobs, so all job descriptions require lifting 50 lbs. above one's head as an essential function, to discourage applicants with disabilities. The two of them believe an expert consultant will protect them from a potential discrimination lawsuit. Answer the following: 1) What legislative and/or ethical standards apply; 2) what decision-making model might you use; and 3) what decision might you arrive at as to if you can help Bob and Mr. Sam out?

Standards relevant to the practice of counseling: Courts, legislation. State boards, certification boards, professional organizations, professional training accreditation (Ethics powerpoint internship modules) Rationale guidance for the ethics process. --Courts: discrimination is illegal --Legislation: not sure --State boards: not sure --Certification board and professional org : promote independence and quality of life. Beneficence. Do not harm. Virtue: --Look at who people are and how to deal with dilemma -----From their perspective, wed be saving them money -----Helping people from further injury/harm -----Protecting people from discriminatory employer -----Taking away opportunities from fully capable people -----Participating in discrimination -----ACA code expressed not to do any harm I ethically cannot help Bob because courts say that we can't discriminate and my ethics require me to do no harm.

There is debate about the best approach to vocational rehabilitation for persons with substance abuse and addiction disorders. On the one hand, some advocate an "abstinence" approach and prefer clients be substance free before job placement efforts can occur. Others suggest a "harm reduction" approach in which clients receive job placement services despite on-going relapses in treatment for substance misuse. Describe the impact of substance abuse on the vocational rehabilitation process. Create an argument for one approach or the other with support from your readings or coursework.

Substance abuse during VR services can have a multitude of impacts. If a client shows up to a meeting and is not sober, that can affect their ability to provide consent and sign important documentation. If they are unable to remain sober for meetings, they probably would struggle to maintain sobriety on the job. Ethically, I would not be able to place someone in a job if both the client and I know they are not capable of remaining sober at work. This creates a huge liability and would erase all the work they put into obtaining the job in the first place. However, the process of getting sober is hard, and expecting complete sobriety is not necessarily realistic. When a client discloses active substance abuse or addiction, it's important to understand where they are at. Is being sober for meetings/work related activities realistically possible for them? I would make it clear that if they truly want a job, it is important to understand why sobriety at work is necessary. I would make them verbally promise to always be sober when participating in anything related to VR or work. It would be made clear that if at any point, they show up intoxicated in any way, we will not be able to continue providing services to them. If they cannot agree, we would close their file or put it on hold until they are confident in their ability to do that. If they feel confident, then services would continue as normal. Everyone deserves to have access to opportunity and many, many people responsibly utilize substances in their personal time. What a client does in their personal time is not our business, until it affects the work we are doing together.

Substance abuse is a significant problem in the United States. Define substance abuse and define addiction. Compare and contrast the medical and social models of substance abuse. How do these models impact the way communities respond to persons who abuse substances?

Substance abuse is overusing a substance or using it for reasons other than its intended purposes. Substance abuse doesn't always lead to addiction, but it is highly probable. The word describes the wide range of disorders; focuses more on the idea of overconsumption of a substance. Addiction is a maladaptive, problematic relationship with a substance that continues despite increasingly negative consequences. A physiological dependence (physical reaction within the body to crave a substance in order to maintain a certain level of functioning) is not needed in order to be classified as an addiction, but can be present.emphasizes the use of substances regardless of the consequences. The pathological craving for the substance. Addiction evolves into more than just merely "liking" or "wanting" the substance. Both tend to be used interchangeably; addictions tend to represent the worse of the cases. The medical model of substance abuse considers this to be a progressive illness that can result in death if not accurately treated with the correct medical intervention. The chemical makeup of the brain is altered which leads to the abuse of substances. Relapse is almost inevitable with this model of substance abuse. This is different than the social model which states that environmental factors influence the individuals proneness to substance use. This can include growing up in the presence of substances, cultural beliefs, and availability and accessibility. These models are similar in the sense that there is something among the individual that increases their risk of substance use, whether that be genetic makeup or social surroundings. With the medical model, substance use is considered a disease. So even if an individual tried to quit on their own, they would struggle and still need medical intervention for a safe recovery process. Because of this, the community responds to these individuals similarly to an individual that may have a broken leg or seizure disorder. They understand that it is not their fault and that they need help just like anyone else. With the social model, the community believes that substance abuse is a choice and they should be able to quit on their own. When an individual either doesn't quit or continues to relapse when attempting to quit, the community believes that it is the individual's fault that they are not succeeding. It is due to a lack of strength and effort, not a lack of professional help. Emphasizes societal and environmental barriers to disabilities/substance use. Viewed as the results of restrictions imposed on society. Emphasized society's failure to take limitations into account.

The Americans with Disabilities Act (ADA) requires accommodations for disability. Define who is eligible for accommodation under ADA legislation. What organizations or businesses are required to comply with the ADA?

The ADA considers the definition of disability to be having a physical or mental impairment that substantially limits a major life activity. Even if the individual has a history of a disability or the employer believes an individual has a disability, then they are eligible for accommodations under the ADA. All public organizations and businesses are required to comply with the ADA. This includes state and local governments, commercial and public accommodation facilities like restaurants, theaters, museums, factories, warehouses, etc., and transportation facilities. When constructing or altering facilities, all businesses, even those that do not serve the public, must comply with new accessible design standards. Federal facilities and religious entities are exempt.

Bob has a disability and he feels that his employer is not offering accommodations under the ADA. Be sure to: 1) describe the ADA; and 2) describe steps that are within your scope of practice that you can take to help Bob. If you find that Bob is not being accommodated, can you suggest to Bob that he sue his employer?

The ADA is the Americans with Disabilities Act that allows individuals with disabilities to request reasonable accommodations in the work environment in order to have opportunity for safe and effective performance for their required job functions. This law not only protects individuals in the workforce, but also in other areas of public life like accessible restrooms, entrances, walkways, etc. It promotes equal opportunity and self-sufficiency. It's a civil rights law that prohibits discrimination against individuals with disabilities in all areas of public life, including jobs, schools, transportation, and all public and private places that are open to the general public. As a rehab counselor, it is within my scope of practice to work with the employer to help them better understand how implementing these accommodations will be beneficial for the work environment as my client can perform the essential functions of his job with the employer's help by applying to ADA law. It is also important to note that for many accommodations implemented, the cost is minimal, which may be helpful in changing his employer's attitude about the situation. I can also ensure that I can work collaboratively with them to move towards a more inclusive workplace. I cannot suggest Bob sue his employer because it is outside my scope of practice. A vocational rehabilitation counselor can play an essential role in facilitating the interaction between an applicant/employee and an employer. The counselor can assist in building employer confidence that a qualified individual with a disability can perform the essential functions of a job with or without a reasonable accommodation. The counselor can also help clients to build skills in how to advocate for themselves. If the employer still denies accommodations, we can help the client retrieve medical documentation of their disability and their doctor ordered limitations. If the employer's barrier for saying yes is financial, VR can pay for necessary accommodations. If the employer still says no, we can do an official ADA check of the site and connect the client to the ADA / and ADa advocate who better understands the laws and regulations surrounding this. I cannot suggest that Bob sue his employer because it is out of my scope of practice.

Choose one (1) theory of individual counseling. Describe the basic philosophy, intervention techniques, and outcomes of your chosen theory. Write a brief critique of the theory that describes the limitations and controversies that may influence your decision to use this theoretical approach with a client or student.

The basic philosophy behind Cognitive-Behavioral Theory is that it isn't the things in our environment or events and situations that we experience that causes stress, anxiety, and other emotional states. What matters is how individuals choose to view those experiences. The way the world is perceived is triggering emotional upsets. The beliefs we have about an experience alters the way we behave, and that will always cause an emotional consequence, whether its positive or negative. One intervention technique is identifying the extreme or irrational thinking. Identifying statements that have "should, must, or ought" shows that an individual believes something should be done a certain way, even if that isn't the right way. Reviewing how negative emotional states are maintained by the cognitive pattern shows how a negative state continues to follow their irrational thoughts. Lastly, modification of immediate thought patterns and generalized longer-term beliefs instill long term adjustment. Outcomes of cognitive-behavioral theory are that individuals will learn to challenge their fears and discomforts as well as gaining confidence through changing their irrational thoughts to rational thoughts and behaviors. One limitation to this approach is that without constant intervention or homework for clients to do at home, it is possible that the effectiveness will decrease and will only be a short-term fix. Some clients may not be equipped or strongly motivated to continue treatment on their own and a relapse may not be something I would consider jeopardizing. In rehab counseling, we have been told many times that family needs to be involved and impacts the individuals success. This approach is individualized and doesn't consider environmental stressors or supports like family and friends. Because of that, this would influence my use of the approach if I knew my client background and found that family impacts their life. Cognitive-behavioral approach also doesn't consider that some negative experience are actually negative and are valid to be irrational. This would influence my use on clients with any past trauma or PTSD.

The concept of dual relationships and ethical standards in counseling related to entering into such relationships have evolved in recent years. First, define, compare, and contrast the terms of "dual relationship" and "boundary crossing." Second, provide the implications of these two (2) terms in your work as a professional counselor.

The ethical standards evolution noted in Erford (2018) p. 74: Reflect times when a counselor needs to of beyond the conventional counseling boundaries for the good of the client but must first take all the appropriate professional precautions such as consultation, informed consent, and documentation. Dual relationship: any relationship between a counselor and client that happens outside of the counseling relationship (personal, professional, blood ties). These types of relationships should be avoided as it may be seen as harmful to the client -Ex: Counseling a co-worker or a consultant. -Implications: don't do this; watch out in rural areas. This can cause harm, so it's best not to find grey area here. Boundary crossing: a deviation from typical therapeutic activity (by either counselor or client) that's harmless, non-exploitative, and possibly supportive of the therapy itself, (--but in the grey area of the code of ethics--) -Ex: Accepting gifts (under $25 in value) from a client who comes from a culture where giving a small gift is a sign of care and respect. If the counselor were to refuse this client's gift, they may harm the client by offending their cultural beliefs and unintentionally hamper the therapeutic process -Implications: Only do this after taking the prof precautions and only if it is beneficial/necessary for the therapeutic relationship/process. May have cultural or other situations. Similarities: Activity beyond the typical therapeutic process Differences: -Dual relationships are considered automatically unethical and consist of having a counseling relationship as well as another type of relationship (friend, coworker, consultant, relative, etc.) -Boundary crossing is not always unethical, and it includes the use of professional precautions such as consultation, informed consent, and documentation.

Briefly outline the five stages of the clients' readiness for change model. Additionally, explain how counselors may aid and impact clients at each of these stages to promote clients' progress.

The first stage is precontemplation where the individual is not considering changing at all. It is possible that they are in denial or have had unsuccessful attempts at change in the past and so they have given up. In this stage counselors can educate the individual on the pros vs cons in making this decision and show them examples of positive outcomes. The second stage is contemplation where the individual has mixed feelings about making changes. They will consider the pros and cons in the aspects of change including time, effort, cost, etc. In this stage counselors can address any concerns the individual may have and provide as many support systems that may benefit the individual. In the third stage, preparation, the individual is preparing by making small changes in their life, but hasn't dove in 100%. During this time a counselor can help the individual develop realistic and achievable goals, as well as create a timeline for change. In the fourth stage, action, the individual is all in and has taken clear steps to make changes in their life. A counselor can provide positive reinforcement during this stage with each accomplishment and goal that has been achieved. The final stage is maintenance and relapse prevention where the individual tries to maintain the new behavior and changes they have made long-term. During the maintenance phase, continued support and encouragement can be provided by a counselor to keep their client on their course to achieve their goals.

Compare and contrast the concepts of multicultural counseling competency versus multicultural orientation (i.e., cultural humility). Which approach is more useful in working with diverse clients?

The goal of cultural competency is to build an understanding of minority cultures to better and more appropriately provide services. Cultural humility puts an emphasis on personal growth, self-reflection and one's personal biases and attitudes towards cultural contexts and awareness. While CC values knowledge and training, CH values introspection (internal reflection of attitudes and biases) and co-training (external and socially applicable training and experience). Shortcomings: CC thinks one can become "certified" in another culture after enough academic learning/training. CH thinks one can never be "certified" in another culture despite years of personal experience in addition to academic knowledge and training. CH's learning is continual, never ends. Strengths: CC provides an end goal and promotes skill building. CH encourages lifelong learning with an appreciation of the journey of growth and understanding. CH also puts professionals and clients in a mutually beneficial relationship (professional can increase their cultural awareness from client/through interactions with client as well as the client learning from the professional providing therapeutic aid) that attempts to diminish the damaging power dynamics. Most Useful: Both concepts are essential to working with diverse clients, and both should be use simultaneously. Personal reflection should be a continuous cycle while meeting with diverse clients, but in order to be prepared for those meetings counselors should feel confident in their cultural competency without going so far as believing that they can become fully competent in a culture outside of their own.

What is the relationship between the research process and the counseling practiced? Why do rehabilitation counselors (RC students), clinical mental health counselors (CMHC students), or school counselors (SC students) need to become research consumers?

The integration of research into practice through an EBP approach to counseling brings out the best elements of practice, clinical experience and reliable treatment protocols together to serve the task of helping clients with the complex problems they bring to counseling Research established efficacy of individual, group, and family counseling for a variety of presenting client concerns. ----Important thing to remember in terms of practice à research now points to a number of very stable trends that support the efficacy of some practices of counseling over others, the differential value of some aspects of counseling over others, and effectiveness of matching certain client problems with specific counseling models. ----These trends inform counseling practice and preparation and form the basis of an EBP model of counseling. EBP has the opportunity to move the profession of counseling out of its theoretical boxes and historical beliefs into an era of integrated practice in which counselors use the best of available science combined with clinical experience to successfully help a wide variety of clients. EBP can provide a source of clinical knowledge that can increase a counselor's effectiveness with clients, become a basis of professional education and counselor development, and serve as a unifying force for the profession that will set the agenda for the next evolution of counseling.

Critique the applicability of trait-factor AND developmental approaches to career counseling when used with persons with developmental cognitive disabilities. Describe an alternative career model that is often employed with consumers with disabilities.

Trait-factor and developmental approaches to career counseling fail to consider the individual outside of the norm or average employed worker. Holland's theory states that personality should match a specific work environment, which should improve job satisfaction. It so simple that it struggles to look past the norm of male vs. female oriented jobs, and categorizes personality often to gender. When considering a consumer with developmental cognitive disabilities, vocational preferences may be unattainable to them. Trait-factor approaches don't focus on individuality as there are only 6 personality types someone could fall into, and someone with a cognitive disability may not be best suited for this type of career counseling. Developmental approaches consider career development over time and does take into account personal, situational and environmental barriers, however it fails to include socioeconomic variables. Individuals with developmental cognitive disabilities are already getting paid significantly less on average than their non-disabled colleagues. These approaches to career counseling fail to consider one of the major struggles with employment that individuals with disabilities face. Social Cognitive Career Theory's (SSCT) choice model is often used with consumers with disabilities. This model takes a consumer's initial choice and divides it into different parts, which are expressions of a primary choice to enter a particular field, taking actions designed to implement that goal, and performance experiences and attainments that affect the shape of one's future. This is important as even though a choice is being made, the environment still plays a role in choices through judgments and experiences. Unlike other models, SSCT choice model takes into account individual factors like background influences, financial support, disabilities, self-efficacy, and any other barriers or experiences that make people different from one another. This model takes self-efficacy and as a guide to working with individuals with disabilities and strengthens it because self-efficacy plays a role in career development.

Discuss microcounseling skills as they relate to trust building. Provide a rationale for the use of specific microskills especially well suited for use in the early phase of a counseling relationship.

Trust building is necessary in order to build a strong therapeutic alliance. The first few sessions with a client are vital for this. This occurs by showing empathy. If unable to show empathy, showing a desire to understand is vital.This creates a safe space for the client to gain comfortability to speak freely. This occurs through asking the right questions and utilizing active listening skills, as well as non verbals. Being respectful of your client and their desires, self-disclosure when relevant and beneficial, and admitting to my own mistakes/shortcomings that show up in the session are also important skills related to trust building. The only way your client will have respect for you and the counseling process is if I model that respect towards them. Self-disclosure is a great tool when used appropriately, because it humanizes the counselor and relaxes the client a bit. Finally, if I am at fault, don't hide the wrongdoing or place the blame on someone else. Pause what I am working on and take the time to apologize. Openly communicate what occurred and have a prepared plan for follow-up. Confessing and correcting shows that I have the client's best interest in mind. These skills, when used successfully, build the foundation for a positive counseling relationship.


Kaugnay na mga set ng pag-aaral

Chapter 14 The Brain and Cranial Nerves (by BuGay)

View Set

كتاب أهم ألف كلمة إنجليزية

View Set

New Testament Survey Unit 7 Test

View Set

Light Independent Reactions in Photosynthesis Pre-Test answer keys

View Set

Topic 4: Ionic + Metallic Bonds, Physical Properties

View Set